You are on page 1of 112

1

Unit

MATRICES AND DETERMINANTS

1.1 INTRODUCTION

matrix is a rectangular array of numbers( Real or Complex) arranged in the form of

A rows and columns. For example,

A= , B= ,C=

Are matrices. Here the numbers are called elements of the matrix. The horizontal lines of
elements are called rows while the vertical lines are called columns.

If in a matrix there are rows and columns it is said to be matrix of order .

The matrices are usually denoted by the capital letters A, B, C, ---.

The elements of the matrix are also enclosed within the brackets of the type
In the above examples A is a matrix of order , B is of order 3 and C is of order .

The matrix is an arrangement of numbers or elements in rows and columns and does not
have any value.

General representation of a matrix:

.
1.2 TYPES OF MATRICES

1. Row matrix – A matrix having only one row and n number of columns is called a row
matrix.
Example: A =
2. Column Matrix – A matrix having m rows and only one column is called a column
matrix..

Example:

3. Zero/Null Matrix – A matrix in which all the elements are zeros is called a zero or a null
matrix and is denoted by O.

Example: O =
4. Square Matrix – A matrix having equal number of rows and columns( say n ) is called
square matrix of order n.

Example: B =

Diagonal elements: In a square matrix the diagonal from left-to-right (↘) is called the
principal diagonal. Entries mii are called the diagonal entries. The others are called
nondiagonal entries

5. Diagonal Matrix - A square matrix in which the non-diagonal elements are equal to zero
is said to be a diagonal matrix.

Example: A =

6. Scalar Matrix - A diagonal matrix in which all the diagonal elements are equal is called
a scalar matrix.

Example: A =

7. Unit Matrix - A scalar matrix in which all the diagonal elements are equal to unity is
called a unit matrix and is usually denoted by I.

Example: I =
8. Transpose of a matrix - If A is a matrix of order then the matrix obtained by
interchanging rows into columns or vice-versa is called transpose of the matrix A which
is of the order and is denoted by AT/A`.
Example:

9. Symmetric Matrix – A square matrix that is equal to its transpose is called symmetric
matrix. (i.e.) .

Example : A = , B=

10. Skew - Symmetric Matrix – A square matrix that is equal to its transpose with negative
sign is called skew-symmetric matrix. (i.e.) .

Example : A = , B= ,C=

11. Upper triangular matrix - A square matrix is called upper triangular if all the entries
below the main diagonal are zero.
Example:

12. Lower triangular matrix - A square matrix is called lower triangular if all the entries
above the main diagonal are zero.
Example:

1.3 ALGEBRA OF MATRICES


1. Equality of matrix
Two matrices A and B are said to be equal if they are of the same order and
corresponding elements are same.
Example: If
Then A = B if and only if, 4 = x ; –2 = y + 4 ; 3= .

2. Multiplication of a matrix by Scalar


Let A be any matrix and K be any scalar then the multiplication of matrix by a scalar K is
obtained by multiplying each element by K.

Example:
A= 2A =

3. Addition / Subtraction of a Matrix


If A and B are the matrices of the same order then A + B is defined to be a matrix
obtained by adding the corresponding elements of A and B.

Example : A + B

Example : A - B
4. Multiplication of Matrices
If A is a matrix of order and B is a matrix of order then the product AB is
defined and is of order .

Example: Let A = and B =

AB =

A= and B =

Example:

1.4 PROPERTIES OF MATRICES

 Distribution Law
o If A and B are two matrices of same order and K is any scalar then
K(A + B) = KA + KB.
o If A, B and C are the three matrices of same order
A(B + C) = AB + AC
(B + C)A = BA + CA

 Commutative Law
If A and B are the two matrices of same order then
o A + B = B+ A

 Associative Law
If A, B and C are the three matrices of same order, then
o (A + B) + C = A + (B + C)
o A.(BC) = (AB).C

 Existence of Identity
If A is any matrix and ‘0’ is a null matrix, then A + 0 = 0 + A = A.

 Matrix multiplication is not commutative in general i.e., .

 If A is a square matrix of order n and I is an identity matrix of same order,


AI = IA = A.

Examples:

1. A = , B=

 A+B = 0

2. If A = , B= , then find A + AB.

 AB =
=

A + AB = +

3. If A = and B = , then find 2A – 3B.

 2A = 3B =

2A – 3B =

4. Find the matrix X such that 3A + 5B +2X = 0 and A = ,B= .

 3A + 5B + 2X = 0
2X = - [3A + 5B]
X = -1/2 [3A + 5B]

3A = , 5B =
3A + 5B = +

X = -1/2

5. If A = then find .


=

6. If A = and B = , Verify = + .

 A+B= +

= , =

+ = + =

= + Hence Proved.
7. If A = Prove that +4 –A 12I = 0.

 =

= =

+4 –A 12I = + - -

= - -

= -

= = 0. Hence Proved
8. Solve for , + = .

 =

and
9. Find a, b, c given = .

 a+3=2 3a – 2b = -7 + 2b
a=2–3 3 (-1) – 2b = -7 + 2b

a = -1 -3 -2b = -7 + 2b
-3 – 4b = -7
-4b = -7 + 3
-14b = +4
b=1

-3a – c = b + 4
-3 (-1) –c = 1+4
3–c=5
-c = 5 – 3
-c = 2
c = -2
a= , b = 1, c =

10. Find and if = .

 =

Solving above equations , and .


11. If A = and B = , Prove that .

- = - = (1)

A+B = + =

A–B= - =

(A+B) (A-B) =

= (2)

from (1) and (2)

12. If =0

 =0

=0

=0
=0

13. Find the value of the matrix C such that 2A + C = 3B where,

A= ,B= .

 Given, 2A + C = 3B

C = 3B-2A

3B = 3 =

2A = 2 =

C = 3B - 2A

C=

C =

14. Find AB and BA, if A = and B = .

 AB =

BA =

= =
15. Find AB and BA, if A = ,B= .

 AB = =

BA = =

16. Find AB and BA if, A = and B = .


AB =

= =

BA =

17. Find AB and BA if, A = and B =

 AB = =

BA = = =
Exercise

 2 4 1 0  2 2
1. If A =   and B = 2 1 find A - B - 2AB.
  1 2   
 4 2 
 2 5 1 6 3  , C = 1 
2. If A =  0 1 3 , B =   3 then verify A(BC) = (AB)C.
   1 0   
 2  1
3. If A =   show that A2 - 4A + 3I = 0.
 1 2 
 3 1
 2 5 prove that A – 8A + 13I = 0.
2
4. If A =
 
2 6 
   3 4 1
5. If A = 5 4 and B =   find (AB)T .
 
1 7  9 11 3

2 6 
   3 4 1
6. If A = 5 4 and B =   find BTAT.
 
1 7  9 11 3

5 3 1  1 7 5 
7.
  
If A + B = 2 4 6 and A- B = 0 2 4 find A and B.

   
1 0 2  5 4 2 
5 3 1  1 7 5 
8.
  
If A - B = 2 4 6 and A + B = 0 2 4 find A and B.

   
1 0 2  5 4 2 
2 3 1  4 6 1
9. If 2A + B =   and 3A + 2 B = 2 3 5 find A and B.
1 4 0   
 2 1 3 1 0 2 
0 2 6  , B = 3 1 4  find AB and A B.
T T
10. If A =
   
5 3 1  1 7 5 
   
11. If A = 2 4 6 and B = 0 2 4 find AT BT.
   
1 0 2  5 4 2 
5 3 1  1 7 5 
   
12. If A = 2 4 6 and B = 0 2 4 find (BA)T.
   
1 0 2  5 4 2 

 x  y z  1 4 
13. If 
   0 6  find x, y, z and w.
 2 x y w   
 x  y z  4 1 
14. If 
   9 6 find x, y, z and w.
 2 x y w   
2 1 0  x 

15. If  x 4 1 1 0 2
  4   0 then find x.
  
 0 2 4   1
1.5 DETERMINANTS
Every square matrix is associated with a real number called its determinant. If A is a square
matrix, its associated determinant is denoted by |A| or det A.

Determinant of order two

Consider a square matrix A = , then the real number ( ) is called the


value of the determinant A.

Example:

1. A =

|A| = 2(0) – (-1)(1) = 1

2. A =

|A| = 4 – 6 = - 2

3. A =

|A| = = 6 - (-1) = 7
Determinants of order three

Consider a square matrix, A = , then the real number associated is,

|A| =

NOTE:

While expanding the determinant with the help of elements of the first row, we have to
put the signs +, , + before the elements respectively.

Example:

1. A=

 |A| = 6(15 – 0) – 2(12 – 0) + 7(4 – 10)

= 6(15) – 2(12) + 7(-6)

= 90 – 24 – 42

= 24

2. A =

 |A| =

= 2(–5) – 1(15 – 0) + 3(6)

= – 10 – 15 + 18

=–7
Evaluate the following :

1.

 |A| = 8 + 3 = 11

2.

 |A| = 18 – 20 = – 2

3.

 |A| = 20 – 24 = – 4

4. Solve for , =

 x2 + 12 = 7 - 6x

x2 + 6x + 5 = 0

x2 + 5x + x + 5 = 0

x (x + 5) + 1(x + 5) = 0

(x+5) (x+1) = 0

x= ;x=

5. Evaluate :

 |A| = 3(-18 - 3) + 5(63 - 4) + 6(-21 - 8)

= 3(-21) + 5(59) + 6(-29)

= -63 + 295 - 174

= 58

6.

 |A|= a(bc – a2) – b(b2 - ac) + c(ab – c2)

= abc – a3 – b3 + abc + abc –c3 + 3abc – a3 – b3 + c3


= a3 + b3 + c3 – 3abc

7. Solve for x, =0

 x(5x - 2x) - 2(2x + x) - 1(4 + 5) = 0

x(3x) -2(3x)-1(9) = 0

3x2 - 6x – 9 = 0

x2 - 2x – 3 = 0

x2 - 3x + x – 3 = 0

(x - 3)(x + 1) = 0

x = 3; x = 1

8. =0

 2(7x + 48) – 3(-28+40) + 4(-24-5x) = 0

14x + 96 – 3(12) + 4(-24 -5x) = 0

14x + 96 – 36 -96 -20x = 0

-6x -36 = 0

6x + 36 = 0

x+6=0

x = -6

1.6 CRAMER’S RULE


Cramer’s Rule for solving a system of two linear equations in two unknowns
ax  by  s
The solution to the system of equations  is given by
 cx  dy  t
s b a s
t d D c t Dy
x  x and y  
a b D a b D
c d c d
a b
Provided that D   ad  bc  0 .
c d

Cramer’s Rule for solving a system of three linear equations in three unknowns
 a1 x  b1 y  c1 z  d1

The solution to the system of equations a2 x  b2 y  c2 z  d 2
a x  b y  c z  d
 3 3 3 3

with
a1 b1 c1 d1 b1 c1 a1 d1 c1 a1 b1 d1
D  a2 b2 c2  0 , Dx  d 2 b2 c2 , Dy  a2 d2 c2 , and Dz  a2 b2 d2
a3 b3 c3 b3 d3 c3 a3 d3 c3 a3 b3 d3
is given by
D Dy D
x x , y , and z  z .
D D D

Example:

Solve using Cramer’s Rule.


2x  3y  5
1.
x  4 y  1

5 3 2 5
Dx 1 4 17 Dy 1 1 5
 x   y  
D 2 3 11 D 2 3 11
1 4 1 4

4x  2y  7
2.
x  5y  1

7 2 4 7
Dx 1 5 37 37 Dy 1 1 11 11
 x    y   
D 4 2 18 18 D 4 2 18 18
1 5 1 5
Exercise:

Solve using Cramer’s rule :


1.  x  y  3z  3 , 2 x  y  z  2 , 3x  4 y  5 z  4 .
2. 7 x  y  z  14 , 2 x  y  z  8 , 3x  y  2 z  8
3. 3x  2 y  2 z  1, 2 x  7 y  3z  12 , 4 x  2 y  z  3 .
4. 9 x  y  2 z  68 , 3x  2 y  3z  10 , 2 x  4 y  3z  27 .
5. 7 x  3 y  z  17 , 6 x  2 y  7 z  65 , x  2 y  3z  18 .
6. 3x  2 y  2 z  19 , x  y  z  8 , x  2 y  3z  16 .
7. 3x  y  2 z  13 , 2 x  y  z  3 , x  3 y  5 z  8 .
8. 8 x  2 y  3z  20 , x  y  4 z  9 , 3x  2 y  z  6 .
9. x  5 y  4 z  5 , 3x  3 y  z  19 , 2 x  3 y  z  7 .
10. 7 x  y  z  4 , 5 x  2 y  2 z  1, 3x  2 y  z  6 .

1.7 PROPERTIES OF DETERMINANTS

1. The determinant of a square zero matrix is zero.


0 0 0
0 0 0 0
0 0 0

2. If all the elements of any row or column of a determinant are zero, then the value of the
determinant is zero.
a1 a2 a3
b1 b 2 b3  0
0 0 0
3. The determinant of a triangular matrix is equal to the product of its diagonal elements.
1 0 0 1 3 7
4 5 0 1 5 6  30 0 4 0 1 4  6  24
3 0 6 0 0 6

4. The determinant of a diagonal matrix is equal to product of its diagonal elements.


x 0 0
0 y 0  x  y  z  xyz
0 0 z
5. If any two rows or columns of a determinant are identical, then the value of the
determinant is zero.
a1 a2 a3
a1 a2 a3  0
b1 b 2 b3

6. If all the rows and columns of a determinant are interchanged then the value of the
determinant is unaltered.

a1 a2 a3 a1 b1 c1
b1 b 2 b3  a 2 b2 c2
c1 c2 c3 a3 b3 c3

7. If any two rows or columns of a determinant are interchanged then the sign of the
determinant is altered but magnitude remains same.
a1 a2 a3 b1 b 2 b3
b1 b 2 b 3   a1 a2 a3
c1 c2 c3 c1 c2 c3
8. If any row or column of a determinant are/ is a multiple of k, where k is a scalar, then the
value of the determinant is also multiplied by k.
a1 ka 2 a3 a1 a2 a3
b1 kb 2 b3  k b1 b 2 b3
c1 kc2 c3 c1 c2 c3

9. If each element of any row or column of a determinant is the sum of two terms, then the
determinant can be expressed as the sum of two determinant.

a1  x a 2 a3 a1 a2 a3 x a2 a3
b1  y b 2 b3  b1 b 2 b3  y b 2 b3
c1  z c2 c3 c1 c2 c3 z c2 c3

10. If to the elements of any row or column, the same multiple of the corresponding elements
of any other row or column are added, then the value of the determinant is unaltered.
a1 a2 a 3 c  c kc a1  ka 2 a2 a3
1 1 1
b1 b2 b3 b1  kb 2 b2 b3
c1 c2 c3 c1  kc 2 c2 c3
Evaluate the following:

1. Δ =

= =0

Δ=0

2. Δ =

= 17*24

Δ=0

3. Δ =

= =0

4. Prove that

Δ= =0

=
=

=0

5. Show that

Δ= = x2(x+6)

R2  R2 – R3

R1  R1 – R3

x2

= x2 (3+x+2) -1(-1)

= x2(5+x)+1

= x2(x+6)
Exercise

a bc 2a 2a
1. Show that 2b bca 2b  ( a  b  c )3
2c 2c c a b
a 2 ab ac
2. Show that ab b 2 bc  4a 2b 2c 2
ac bc c 2
1 a 1 1
1 1 1
3. Show that 1 1 b 1  abc(1    )
a b c
1 1 1 c
x x 2 1  x3
4. Show that 1  xyz  0 , if x, y, z are all different and y y 2 1  y3  0
z z2 1  z3
1 1 1
5. Show that a 2 b2 c 2  (a  b  c)(a  b)(b  c)(c  a )
bc ca ab
1 1 1
6. Show that x2 y2 z 2  ( x  y )( y  z )( z  x)( xy  yz  zx)
x3 y3 z3
1 bc b2  c 2
7. Show that 1 c  a c 2  a 2   a  b  b  c  c  a 
1 a  b a 2  b2
2 3 4
8. Solve 4 x 8  0 .
5 6 7
1 2 3
9. Solve 2 x 3  0.
3 4 3

1.8 INVERSE OF A SQUARE MATRIX


Minor of an element

Let A = be a square matrix of order 3.

Then minor of an element (i = 1, 2, 3, j = 1, 2, 3) is defined to be the second order


determinant obtained by deleting ith row and jth column and is denoted by .
Example:
5 1 2
Let A =
3 2 3
8 1 4 2 3
= minor corresponding to 5 is 11.  11
1 4

1 2
= minor corresponding to the -3 is 2. 2
1 4
1 2
= minor corresponding to the 8 is 7. 7 etc.
2 3
Co-factor of an element

The co-factor of an element in the matrix A is defined as , where is the minor


of and is denoted by .

Thus , i = 1, 2, 3, j = 1, 2, 3

The signs of the co-factors can be remembered as follows:

  
  
  
Adjoint of a Square Matrix

Let A = be a square matrix of order 3.

Then adjoint of A is defined to be the transpose of the matrix formed by the corresponding
co-factors.

where is the co-factor of an for i = 1, 2, 3, j = 1, 2, 3.

Example:
 5 1 2 
 Find the adjoint of the matrix
A   3 2 3 
 8 1 4 
T
 2 3 3 3 3 2 
   
 1 4 8 4 8 1 
 1 2 5 2 5 1 
adj A      
 1 4 8 4 8 1 
 
1 2

5 2

5 1 
 2 3 3 3 3 2 

 11 2 7 
adj A   36 36 9 
 13 13 13 

Inverse of a Square Matrix

Let A be a square matrix. If there exists a matrix B such that AB  BA  I where I is a unit
1
matrix then B is called the inverse of A and is written as B  A1 . i.e. A1  adj A
A
Example :
1 3 2 
Find the inverse of the square matrix A   0 1 2  .
 2 1 3 
1 3 2
 A  0 1 2 9
2 1 3

Since A  0 , A1 exists.


4 2  1 7 4 
T
1
adj A   7 1 5    4 1 2 
 
 4 2 1   2 5 1 
 1 7 4 
  4 1 2  .
1 adj A 1
A
A 9
 2 5 1 

Exercise

Find the inverse of the following matrices:


1 7 4 1 9 4
5 6 1
5.  4 5 2 
2 1 1
1. 2. 3. 4. 4 1 2
3 4 3 0 3 4
2 5 1  2 5 1

1.9 SOLUTION OF LINEAR EQUATIONS USING MATRIX METHOD

a1 x  b1 y  c1 z  d1
Let a2 x  b2 y  c2 z  d 2 be a set of simultaneous equations in x, y, z.
a3 x  b3 y  c3 z  d3

These equations can be expressed in the matrix form as,


 a1 b1 c1   x   d1 
a b c  .  y   d 
 2 2 2    2
 a3 b3 c3   z   d3 

i.e. AX  B ,

 a1 b1 c1   x  d1 
where, A   a2 b2 c2  , X   y  , B   d 2 
  
 a3 b3 c3   z   d3 
Now
AX  B
A1  AX   A1 B
 A A X  A
1 1
B
I X  A1 B
X  A1 B

Therefore on equating the elements of X and A1B we get the values of x, y, z .


Example:
1. Solve the equations using the Matrix Method:
3x  y  2 z  13, 2 x  y  z  3, x  3 y  5z   8 .
 These equations can be expressed in the matrix form as,
 3 1 2   x  13 
 2 1 1 .  y    3 
     
1 3 5  z   8
i.e. AX  B ,

 3 1 2   x 13 
where, A   2 1 1 , X   y  , B   3 
   
1 3 5  z   8

3 1 2
A 2 1 1   5  0
1 3 5
 A1 exists.

Hence
 2 9  2 1 1
T
5 
adj A   1 17 10   9 17 7 
 
 1 7 5   5 10 5 

 2 1 1
1 
9 17 7 
11
A  adj A  
A 5
 5 10 5 
 2 1 1 13 
1 
X   9 17 7   3 
5
 5 10 5   8

 15
1 
X   10 
5
 5 

 x 3
 y    2 
   
 z   1 

Therefore x  3, y   2, z  1 .
Exercise

1. 2 x  7 y  z  40 , 3x  y  4 z  15 , x  5 y  3z  30 .
2. 3x  2 y  7 z  8 , 4 x  2 y  5z  9 , 2 x  5 y  2 z  12 .
3. 7 x  y  z  15 , 2 x  y  z  4 , 3x  2 y  4 z  29 .
4. 2 x  y  6 z  7 , 4 x  3 y  z  8 , x  3 y  5z  9 .
5. 2 x  y  5z  26 , x  y  z  7 , 3x  5 y  2 z  22 .
6. x  2 y  3z  17 , 6 x  y  5z  34 , 2 x  5 y  z  18 .
7. 4 x  y  z  5 , 2 x  5 y  7 z  9 , 3x  7 y  z  8 .
8. x  5 y  4 z  5 , 3x  3 y  z  19 , 2 x  3 y  z  7 .
9. 7 x  y  z  4 , 5x  2 y  2 z  1, 3x  2 y  z  6 .
10. 2 x  y  6 z  22 , 3x  4 y  z  8 , 2 x  3 y  2 z  2 .

1.8 APPLICATIONS OF MATRICES AND DETERMINANTS


1. The following matrix consists of a shoe store’s inventory of flip flops, clogs, and Mary
Janes in sizes small, medium, and large:
The store wants to know how much their inventory is worth for all the shoes. How
should we set up the matrix multiplication to determine this the best way?

Solution:

So we’ll have $1050 worth of small shoes, $2315 worth of medium shoes, and $1255
worth of large shoes for a total of $4620.

2. A nut distributor wants to know the nutritional content of various mixtures of almonds,
cashews, and pecans. Her supplier has provided the following nutrition information:

Her first mixture, a protein blend, consists of 6 cups of almonds, 3 cups of cashews, and 1
cup of pecans. Her second mixture, a low fat mix, consists of 3 cups of almonds, 6 cups
of cashews, and 1 cup of pecans. Her third mixture, a low carb mix consists of 3 cups of
almonds, 1 cup of cashews, and 6 cups of pecans. Determine the amount of protein,
carbs, and fats in a 1 cup serving of each of the mixtures.

Solution:

Sometimes we can just put the information we have into matrices to sort of see what we
are going to do from there. It makes sense to put the first group of data into a matrix with
Almonds, Cashews, and Pecans as columns, and then put the second group of data into a
matrix with information about Almonds, Cashews, and Pecans as rows. This way the
columns of the first matrix lines up with the rows of the second matrix, and we can
perform matrix multiplication. This way we get rid of the number of cups of Almonds,
Cashews, and Pecans, which we don’t need. So here is the information we have in
table/matrix form:
Then we can multiply the matrices (we can use a graphing calculator) since we want to
end up with the amount of Protein, Carbs, and Fat in each of the mixtures. The
product of the matrices consists of rows of Protein, Carbs, and Fat, and columns of the
Protein, Low Fat, and Low Carb mixtures:

But we have to be careful, since these amounts are for 10 cups (add down to see we’ll get
10 cups for each mixture in the second matrix above). Also, notice how the cups unit
“canceled out” when we did the matrix multiplication (grams/cup time cups = grams).

So to get the answers, we have to divide each answer by 10 to get grams per cup. So t
he numbers in bold are our answers:

Exercise

1. The first table below show the points awarded by judges at a state fair for a crafts contest
for Brielle, Brynn, and Briana. The second table shows the multiplier used for the
degree of difficulty for each of the pieces the girls created. Find the total score for each
of the girls in this contest.
2. A florist is making 5 identical bridesmaid bouquets for a wedding. She has $610 to
spend (including tax) and wants 24 flowers for each bouquet. Roses cost $6 each, tulips
cost $4 each, and lilies cost $3 each. She wants to have twice as many roses as the other
2 flowers combined in each bouquet. How many roses, tulips, and lilies are in each
bouquet?
3. A building contract specifies in constructing three types of buildings (a) Classical
Indian (b) Gothic and (c) Hacienda. The main inputs required are steel, wood, cement,
paint and labour. The quantities of inputs required are measured as steel in tones, wood
in cubic feet, cement in tones, paint in kilo litres and labour in number of men. The
information given below is the input requirements for each type of building.

Steel Wood Cement Paint Labour


Classic
5 20 16 7 17
Indian
Gothic 7 18 12 9 21
Hacienda 6 28 8 5 13

The cost of inputs is available as follows: steel Rs. 20000 per ton, wood Rs. 800 per
cubic feet, cement Rs. 18000 per ton, paint Rs. 10000 per kilolitre and labour Rs. 5000
per head. Using matrix operations compute the cost of construction for each type
building. If the contractor desires a profit margin of 10% on cost, how much should he
quote for each type of building?

4. A manufacturing company produces four products A, B, C and D. Each product is made


from raw materials P, Q and R. One unit of A requires 2 units of P, 1 unit of Q and 4
units of R. One unit of B requires 5 units of P and 3 units of R. One unit of C requires 4,
3 and 2 units of P, Q and R respectively and one unit of D requires 4, 1 and 2 units of P,
Q and R respectively. Find in matrix form: (a) the total cost of material consumed (b)
the total production cost (c) total sales and (d) total contribution and contribution per unit
of each product.
5. Olympic Tyres has two showrooms in Bangalore. It markets three types in both the
showrooms. The three types of tyres are Truck Tyres (T), Car Tyres (C) and Scooter
Tyres (S). The sales of the two showrooms I and II for the month of April are given
below in Matrix A:
 10 90 60 
A 
 20 40 40 
The selling prices of the T, C and S are Rs.5000, Rs. 1000 and Rs.300 respectively for
each tyre. The company decided to reduce the tyre prices by 10% per unit so as to
augment its sales in May, because of which the sales in units are expected to increase by
20% from April to May. Compute the expected increase in sales value from April to
May.
An Input Output Problem

Input-output problems are seen in Economics, where we might have industries that produce
for consumers, but also consume for themselves. An application of matrices is used in this input-
output analysis, which was first proposed by Wassily Leontief; in fact he won the Nobel Prize in
economics in 1973 for this work.
We can express the amounts (proportions) the industries consume in matrices, such as in the
following problem:
The following coefficient matrix, or input-output matrix, shows the values of energy and
manufacturing consumed internally needed to produce $1 of energy and manufacturing,
respectively. In other words, of the value of energy produced (x for energy, y for
manufacturing), 40 percent of it, or .40x pays to produce internal energy, and 25 percent of it, or
.25x pays for internal manufacturing. Of the value of the manufacturing produced, .25y pays
for its internal energy and .10y pays for manufacturing consumed internally. The inputs are the
amount used in production, and the outputs are the amounts produced.

(a) If the capacity of energy production is $15 million and the capacity of manufacturing
production is $20 million, how much of each is consumed internally for capacity production?
(b) How much energy and manufacturing must be produced to have $8 million worth of energy
and $5 million worth of manufacturing available for consumer use?
Solution:
(a) If production capacities are $15 million for energy and $20 million for manufacturing, the

amount consumed internally is . So $11 million of energy is


consumed internally and $5.75 million of manufacturing is consumed internally.
This makes sense, for example, since we’re multiplying the proportion of energy consumed
internally (.4) by the production capacity for energy ($15 million) and adding that to the
proportion of energy needed for internal manufacturing (.25) by the production capacity of
manufacturing ($20 million) to get the total dollar amount of energy needed or consumed
internally ($11 million). Then we do the same for manufacturing.

(b) The amount of energy and manufacturing to be produced to have $8 million worth of energy
and $5 million worth of manufacturing available for consumer (non-internal) use is solved using
the following equation (we want what’s “left over” after the internal consumption, so it makes

sense): . To get , we can use the formula

. So the two industries must produce


$17.7 million worth of energy and $10.5 million worth of manufacturing, respectively.
2
Unit

COMMERCIAL ARITHMETIC

2.1 SIMPLE INTEREST

hen an investor lends money to a borrower, the borrower must pay back the money

W originally borrowed and also the fee charged for the use of the money. This fee charged
is called as interest and the capital originally lent by the investor is called the principal.
From the investor’s point of view, interest is the income from invested capital. The sum
of the principal and the interest due is called the amount or accumulated value. Any interest
transaction can be described by the rate of interest, which is the ratio of the interest earned in one
time unit to the principal.

We shall use the following notations:

P  principal
I  simple interest
A  amount or accumulate d value of P
R  rate of interest
N  time period

The simple interest I on principal P for N years at annual rate R is given by

PNR
I
100
and the amount A is given by
PNR  NR 
A PI  P  P 1  
100  100 

 NR 
The factor 1   in Error! Reference source not found. is called an accumulation factor at
 100 
simple interest, and the process of calculating A from P using Error! Reference source not found.
is called accumulation at simple interest. From Error! Reference source not found., we have

1
A  NR 
P  A 1  
 NR   100 
 1  
 100 
1
 NR 
as the present or discounted value at rate R of A due in N years. The factor 1   in
 100 
Error! Reference source not found. is called a discount factor at simple interest, and the process of
calculating P from A using Error! Reference source not found. is called discounting at simple
interest.

The time N must be in years. If the time is given in months, then

number of months
N
12

If the time is given in days, we may calculate exact simple interest or simple interest, on the basis of
a 365-day year (leap year or not), that is,

number of days
N
365

Solved Problems

1) Find the simple interest and the maturity amount on Rs. 20,000 for 6 years at 10 % per annum.

Solution:

Given P  Rs.20,000 , N  6 years , R  10% p.a.

PNR 20000  6 10


Then the simple interest I   = Rs. 12,000.
100 100

Hence the maturity amount A  P  I  Rs. 20,000  Rs. 12,000

 Rs. 32,000

2) Find the simple interest and the maturity amount on Rs. 16,000 for 2 years at 17.5 % per annum.

Solution:

Given P  Rs.16,000 , N  2 years , R  17.5% p.a.


PNR 16000  2 17.5
Then the simple interest I   = Rs. 5,600.
100 100

Hence the maturity amount A  P  I  Rs. 16,000  Rs. 5,600

 Rs. 21,600 .

3) Find the simple interest and the maturity amount on Rs. 1,20,000 for 3 years at 15 2 % per
3
annum

Solution:

47
Given P  Rs.1, 20,000 , N  3 years , R  15 2 % p.a.  % p.a.
3 3

PNR 120000  3  47
Then the simple interest I   = Rs. 56,400.
100 3 100

Hence the maturity amount A  P  I  Rs. 1, 20,000  Rs. 56, 400

 Rs. 1,76, 400 .

4) Find the simple interest and the maturity amount on Rs. 15,000 for 3.5 years at 12 % per annum.

Solution:

Given P  Rs.15,000 , N  3.5 years , R  12% p.a.

PNR 15000  3.5 12


Then the simple interest I  = = Rs. 6,300.
100 100

Hence the maturity amount A  P  I  Rs. 15,000  Rs. 6,300

 Rs. 21,300 .

5) Find the simple interest and the maturity amount on Rs. 18,000 for 3 years and 3 months at 16%
per annum.

Solution:

Given P  Rs.18,000 , R  16% p.a.

3
and N  3 years and 3months  3 years and years
12
1
3 years  3.25 years .
4

PNR 18000  3.25 16


Then the simple interest I  = = Rs. 9,360.
100 100

Hence the maturity amount A  P  I  Rs. 18,000  Rs. 9,360

 Rs. 27,360 .

6) Find the simple interest and the maturity amount on Rs. 35,000 for 146 days at 13% per annum.

Solution:

146
Given P  Rs.35,000 , R  13% p.a. and N  146 days  years .
365

PNR 35000 146 13


Then the simple interest I  = = Rs. 1,820.
100 365 100

Hence the maturity amount A  P  I  Rs. 35,000  Rs. 1,820

 Rs. 36,820 . .

7) Find the simple interest and the maturity amount on Rs. 72,000 for 2.5 years at 14.5% per annum.

Solution:

Given P  Rs.72,000 , N  2.5 years , R  14.5% p.a.

PNR 72, 000  2.5 14.5


Then the simple interest I  =
100 100

= Rs. 26,100.

Hence the maturity amount A  P  I  Rs. 72,000  Rs. 26,100

 Rs. 98,100 .

8) Find the simple interest and the maturity amount on Rs. 1,80,000 for 4 years and 6 months at

12 1 % per annum.
3

Solution:
37
Given P  Rs.1,80,000 , R  12 1 % p.a. = % p.a.
3 3

6
And N  4 years and 6 months  4 years and years
12

1
4 years  4.5 years
2

PNR 1,80, 000  4.5  37


Then the simple interest I  = = Rs. 99,900.
100 3 100

Hence the maturity amount A  P  I  Rs. 1,80,000  Rs.99,900

 Rs. 2,79,900 .

9) Find the simple interest and the maturity amount on Rs. 60,000 for 216 days at 8 1 % per annum.
2

Solution:

17
Given P  Rs.60,000 , R  8 1 % p.a. = % p.a.
2 2

216
and N  216 days  years .
365

PNR 60000 17  216


Then the simple interest I  = = Rs. 3,018.08.
100 2  365 100

Hence the maturity amount A  P  I  Rs. 60,000  Rs.3,018.08

 Rs. 63,018.08 .

10) Find the simple interest and maturity amount on Rs. 1,00,000 for 292 days at 3% per month.

Solution:

Given P  Rs.1,80,000 , R  3% p.m = (3 12) % p.a.  36 % p.a.

292
and N  292 days  days
365

PNR 1, 00, 000  292  36


Then the simple interest I  = = Rs. 28,800.
100 365 100
Hence the maturity amount A  P  I  Rs. 1,00,000  Rs.28,800

 Rs. 1, 28,800 .

11) Find the simple interest and maturity amount on Rs. 1,50,000 for 73 days at 2% per month.

Solution:

Given P  Rs.1,50,000 , R  2% p.m. = (2 12)% p.a.

 24% p.a.
73
And N  73 days  years .
365

PNR 1,50, 000  73  24


Then the simple interest I  = = Rs. 7,200
100 365 100

Hence the maturity amount A  P  I  Rs. 1,50,000  Rs.7, 200

 Rs. 1,57, 200 .

12) Find the simple interest and maturity amount on Rs. 15,000 from June 1 to August 13 at 7% per
annum.

Solution:

Given P  Rs.15,000 , R  7% p.a.

and N = (30 days in June) + (31 days in July) + (12 days in Aug).

73
= 73 days = years.
365

PNR 15000  7  73
Then the simple interest I  = = Rs. 210.
100 365 100

Hence the maturity amount A  P  I  Rs. 15,000  Rs.210

 Rs. 15, 210

13) A depositor placed a sum of Rs. 50,000 into his savings a/c with a bank on 12th March 2004 and
withdraws the entire balance in his a/c on 29th June 2004. The bank rules permit simple interest
at 3.75% p.a. How much did he withdraw from his account?

Solution:

Given P  Rs.50,000 , R  3.75% p.a.


and N = (31-11) days in March + 30 days in April + 31 days in May

+ 28 days in June. (excluding 29th June )

109
= 109 days = . (Since, 2004 is a leap year)
366

PNR 50, 000  3.75 109


Then the simple interest I  = = Rs. 558.40.
100 366 100

Hence the maturity amount A  P  I  Rs. 50,000  Rs. 558.40

 Rs. 50,558.40 .

He withdraws Rs. 50,558.40 from his account.

14) An amount given on simple interest of 13% per year becomes Rs.22,240 after 3 years. What was
the principal amount?

Solution:

Given A  Rs.22, 240 , N  3 years , R  13% p.a.

1
A  NR 
Since, P  = A 1  
 NR   100 
1  
 100 

1
 13  3 
We have, P = 22,240 1   = Rs. 16,000.
 100 

15) An amount given on simple interest of 1% per month becomes Rs. 4,00,000 after 5 years. What
was the principal amount?

Solution:

Given A  Rs. 4,00,000 , N  5 years and R  1% p.m.  (112)% p.a.  12% p.a.

1
A  NR 
Since, P  =A 1  
 NR   100 
1  
 100 

1
 12  5 
We have, P = 4,00,000 1   = Rs. 2,50,000.
 100 

16) An amount given on simple interest of 2% per month becomes Rs. 52,400 after 73 days. What
was the principal amount?

Solution:

Given A  Rs.52,400 ; R  2% p.m.  (2  12)% p.a.  24% p.a.

73
and N  73 days  years .
365
1
A  NR 
Since, P  = A 1  
 NR   100 
1  
 100 
1
 24  73 
We have, P = 52,400 1   = Rs. 50,000.
 365  100 

17) An amount given on simple interest of 16.5% per year becomes Rs.23,662.50 after 3 years and 6
months. What was the principal amount?

Solution:

Given A  Rs. 23,662.50 ; R  16.5% p.a.

6
and N  3 years and 6 months  3 years and years
12

1
3 years  3.5 years .
2
1
A  NR 
Since, P  = A 1  
 NR   100 
1  
 100 
1
 16.5  3.5 
We have, P = (23,662.50) 1   = Rs. 15,000.
 100 

18) In how many years do an amount becomes 4 times itself at 12% per annum simple interest?

Solution:

If P is the principal, then given that A=4P; R  12% p.a.


PNR  NR 
Since, A  P  I  P   P 1  
100  100 

 12 N 
We have, 4 P  P 1  .
 100 

12 N
Canceling P on both sides, we get 4  1  , which on simplifying gives
100

300
N = 25 years.
12

3
19. In what time will the simple interest on any sum of money at 3 % p.a. be 0.09375 of the
4
principal?

Solution:

Let P be the principal.

3 15
Given R = 3 % p.a. = % p.a. and I = 0.09375 P
4 4

PNR 100 I
Since, the simple interest I  , we have N  .
100 PR

100  0.09375 P
Hence, N  = 2.5 years.
15
P
4

20) At what rate of interest will Rs. 3,000 become Rs. 3,600 in 4 years?

Solution:

Given P  Rs.3,000 ; N  4 years and A = Rs. 3600

Hence the simple interest I  A  P = 3600 – 3000 = Rs. 600.

PNR 100 I
Then the simple interest I  and hence R 
100 PN

100  600

3000  4

= 5%
1 3
21. The interest on a certain sum of money at the end of 6 years amounts to of the sum itself.
4 8
What rate percent was charged?

Solution:

Let P be the principal and R be the rate of interest per annum.

1 3
Given N = 6 years and I = P .
4 8

PNR 100 I
Then the simple interest I  and hence R  .
100 PN

3
100  P
Hence, R  8 = 6% p.a.
25
P
4

1 3
22. If the rate of interest is reduced from 4 % to 3 % per year, find the decrease in a half yearly
2 4
interest on Rs. 1,600.

Solution:

1
Given P = Rs. 1,600 and N = year.
2

1 9
When R = R1= 4 % = % p.a., we have the simple interest
2 2

1 9
1600  
PNR1 2 2 = Rs. 36, and
I1  
100 100

3 15
When R =R2 = 3 % = % p.a., we have the simple interest
4 4

1 15
1600  
PNR2 2 4 = Rs. 30.
I2  
100 100

Hence the decrease in a half yearly interest = Rs. 36 – Rs. 30 = Rs. 6.

23. A man borrows Rs. 50,000 on simple interest at the rate of 6% per annum. After 3 years he
pays back Rs. 17,700, how much he has to pay after 5 years to clear the loan?
Solution:

Given P  Rs. 50,000 ; R = 6%.

Amount repaid after 3 years = Rs. 17,700

17700
Therefore, the corresponding principal = = Rs. 15,000.
 (3)(6) 
1  
 100 

Principal amount left for 


 = Rs. 50,000 – Rs. 15,000
the remaining 2 years 

= Rs. 35,000

PNR 35,000  5  6
Then the simple interest I  = Rs. 10,500.
100 = 100

Amount to be repaid
 = Rs. 35,000  Rs.10,500
after 5 years 

 Rs. 45,500 .

24) Divide Rs. 9000 into two parts such that simple interest on one at 5% for 4 years may be
double to that on other at 3% for 5 years.

Solution:
Let one part of Rs.9000 be x.

Then the remaining part is Rs. 9000 – x.

Let I1 be the interest on Part I and I2 be the interest on Part II

i.e., with the given data, we have

Part I Part II
P1=Rs. x; N1 = 4 years P2 = Rs. (9000 – x); N2 = 5 years
R1=5% p.a. ; I1 = 2I2 R2 = 3% p.a.
P1 N1 R1 P2 N 2 R2
I1  2I 2  I2 
100 100
x(4)(5) (9000  x)(5)(3)
2I 2  ------(1) I2  ------(2)
100 100
Substituting (2) in (1) we get,
(9000  x)(5)(3) x(4)(5)
2 
100 100 ,

which on simplifying, gives (9000  x)(3)  2 x

(i.e.) x  Rs. 5, 400 .

Therefore the two parts of Rs.9,000 are Rs. 5,400 and Rs. 3,600.

25) A certain sum lent at simple interest amounts to Rs.1320 in 4 years and the same sum
amounts to Rs. 1400 in 5 years, rate of interest being the same. Find the sum and rate of
interest.

Solution:

From the given data, we have

Case I Case II
P2 = Rs. x; N2 = 5 years; A2 =
P1=Rs. x; N1 = 4 years; A1 = Rs. 1,320
Rs.1,400
 N R 
 NR  A2  P2 1  2 2 
A1  P1 1  1 1   100 
 100 
 (5) R2 
 (4) R1  1400  P2 1  
1320  P1 1   -------(1)  100 
------
 100 
(2)
Also given that P1 = P2 = P (say) and R1 = R2 = R (say). Therefore

Dividing (2) by (1), we get

 (5) R2   (5) R 
1   1  
1400 P2  100   100 
 
1320 P1  (4) R1   (4) R 
1   1  
 100   100 

1400 1  (0.05) R 

1320 1  (0.04) R 
(i.e.)

Simplifying, we get R = 8% p.a.

Substituting R = 8% p.a. in (1) or (2) we get P = Rs. 1,000.

26. A sum of Rs. 7500 is annually deposited (at the beginning of the year) in a bank for 4 years.
If the bank allows simple interest on the deposits, what amount will be standing at the credit
of the depositor at the end of 4 years? Bank allows interest at the rate of 5% p.a.
Solution:

Let P be the sum deposited in the bank at the beginning of each year.

Given P = Rs.7,500 and R = 5% p.a. for each year.

For the first year deposit, the duration is 4 years.

For the second year deposit, the duration is 3 years.

For the third year deposit, the duration is 2 years.

For the fourth year deposit, the duration is 1 year

Then,

 N R  N R  N R  N R
A  P1  1   P1  2   P1  3   P1  4 
 100   100   100   100 

 (1)(5)   (2)(5)   (3)(5)   (4)(5) 


A  P1    1    1    1  
 100   100   100   100 

 105   110   115   120 


A  7500    
 100   100   100   100 

A  7500 (4.5)  Rs. 33,750.

27) What will be the amount in 6 years at 7% simple interest per annum of a sum whose amount
in 2 years at the same 7% rate of interest is Rs. 59,287?

Solution:

Let P be the principal.

Case I:

Given N1 = 6 years and R1 = 7% p.a.

 N1 R1 
Let A1 be the maturity amount, then A1  P1  .
 100 

 67 
Hence A1  P1    P1.42 . -------------(1)
 100 

Case II:

Given N2 = 2 years, R2 = 7% p.a. and A2 = Rs. 59,287.


 N R   27 
Hence A2  P 1  2 2  , implies 59,287 = P1   , which on simplifying gives
 100   100 
59,287= P(1.14) . Hence

59280
P  Rs. 52,000 .
1.14

Using P = Rs. 52,000 in (1) we get A1=P (1.42) = 52,000 (1.42)

= Rs.73,840.

28. What sum of money lent at simple interest of 5.5% per annum for 4 years will produce same
amount as Rs. 5,500 lent at 4% per annum, for 3.5 years?

Let P be the principal.

Case I:

Given N1 = 4 years and R1 = 5.5% p.a.

 NR 
Let A1 be the maturity amount, then A1  P 1  1 1  .
 100 

 4  5.5 
Hence A1  P1    P1.22 . -----------(1)
 100 

Case II:

Given N2 = 3.5 years, R2 = 4% p.a. and A2 = Rs. 5,500.

 N 2 R2  3.5  4 
Hence A2  P1   implies A2= 55001   = Rs. 6,270. ---(2)
 100   100 

Given that A1 = A2. Therefore, using (1) and (2), we have

A1=P (1.22)= 6270 = A2

Simplifying, we get P = 6270 = Rs. 5139.34.


1.22

Supplementary Problems
1) Find the simple interest and maturity amount on
a) Rs. 5,000 for 4 yeas at 8% p.a.
b) Rs. 7,000 for 3 years at 6% p.a.
c) Rs. 4,500 for 6 years at 14.5% p.a.
d) Rs. 3,500 for 4 years at 12 2 % p.a.
3
e) Rs. 22,000 for 6 years and 9 months at 18 % p.a.
f) Rs. 45,000 for 3 years and 2 months at 6.2 % p.a.
g) Rs. 1,35,000 for 4 years and 7 months at 11 1 % p.a.
3
h) Rs. 55,000 for 8 years and 9 months at 3 % p.m.
i) Rs. 62,000 for 2 years and 4 months at 1.5% p.m.
j) Rs. 18,000 for 73 days at 2 1 % p.m.
3
k) Rs. 92,000 for 292 days at 14% p.a.
l) Rs. 5,000 from May 2 to July 14 at 7% p.a.
m) Rs. 12,000 from 10th January to 11th March in a leap year at 6% p.a.
n) Rs. 42,000 from 6th January 2004 to 7th July 2004 at 13.5% p.a.
2) An amount given on simple interest of 9% per year becomes Rs. 7620 after 3 years. What was
the principal amount?
3) An amount given on simple interest of 12% per annum becomes Rs. 86,400 after 5 years. What
was the principal amount?
4) An amount given on simple interest of 1% per month becomes Rs. 44,000 after 10 years. What
was the principal amount?
5) An amount given on simple interest of 2% per month becomes Rs. 87,680 after 146 days. What
was the principal amount?
6) An amount given on simple interest of 11.5% per annum becomes Rs. 56,100 after 3years and 6
months. What was the principal amount?
7) In how many years do an amount becomes 3 times itself at 5% simple interest?
8) In how many years do an amount doubles itself at 8% simple interest?
9) At what rate of interest will Rs. 15,000 become Rs. 25,500 in 10 years?
10) At what rate of interest will Rs. 6500 becomes Rs.9100 in 8 years?
1 7
11) The interest on a certain sum of money at the end of 3 years amounts to of the sum itself.
2 8
What rate percent was charged?
1 1
12) If the rate of interest is reduced from 7 % to 5 % per year, find the decrease in a half yearly
2 2
interest on Rs. 10,000.
13) If the rate of interest is reduced from 6 2 % to 3 3 % per year, find the decrease in a half yearly
5 5
interest on Rs. 12,500.
14) A man borrows Rs. 1,00,000 on S.I. at the rate of 10% per annum. After 4 years he pays back
Rs. 70,000, how much he has to pay after 10 years to clear the loan?
15) A man borrows Rs. 25,000 on S.I. at the rate of 13.5% per annum. After 4 years and 3 months he
pays back Rs.15,737.50, how much he has to pay after 9 years to clear the loan?
16) Divide Rs. 50,000 into two parts such that simple interest on one at 8% per annum for 5 years
may be double to that on the other at 6 % per annum for 6 years.
17) Divide Rs. 1,00,000 into two parts such that simple interest on one at 10 % per annum for 10
years may be double to that on the other at 7 % per annum for 15 years.
18) Divide Rs. 20,000 into two parts such that simple interest on one at 8.5% per annum for 6 years
may be triple to that on the other at 6.5 % per annum for 10 years.
19) A certain sum lent at simple interest amounts to Rs. 1320 in 4 years and the same sum amounts to
Rs. 1400 in 5 years. Find the sum and rate of interest.
20) What will be the amount in 6 years at 7% per annum simple interest of a sum whose amount in 2
years at the same 7% per annum rate of interest is Rs. 19,380.
21) What sum of money lent at simple interest of 3.5 % per annum for 5 years will produce same
amount as Rs. 3,000 lent at 5% per annum, for 4.5 years ?
22) A sum of Rs. 2,40,000 is lent in 2 parts, I part is lent at 6% for 4 years and II part is lent at 9%
per annum for 3 years. If the total income earned is Rs. 61,800 find the sums lent at each rate.
23) In what time will the simple interest on any sum of money at 3 3 % per annum be 0.09375 of the
4
principal?
1
24) A man lent equal sums of money at 5 % and 4% per annum respectively for a period of 3 years.
2
1
If he earned Rs. 72 more from the money lent out at 5 %. Find the sum of money lent at 4 %.
2
Answers:
(1) (a) Rs.1,600; Rs.6,600 (1) (b) Rs.1,260; Rs.8,260 (1) (c) Rs.3,915; Rs.8,415
(1) (d) Rs1,773.33; Rs.5,273.33 (1) (e) Rs.26,730; Rs.48,730 (1) (f) Rs. 8,835; Rs.53,835
(1) (g) Rs.70,125; Rs.2,05,125 (1) (h) Rs.1,73,250; Rs.2,28,250 (1) (i) Rs. 26,040; Rs.88,040
(1) (j) Rs. 1008; Rs.19,008 (1) (k) Rs.10,304;Rs.1,02,304 (1) (l) Rs. 70 ; Rs.5070
(1) (m)Rs.120; Rs.12,120 (1) (n) Rs.2,842; Rs.44,842.
(2) Rs. 6,000 (3) Rs.54,000 (4) Rs.20,000
(5) Rs.80,000 (6) Rs.40,000 (7) 40 years
(8) 12.5 years (9) 7% p.a. (10) 5%
(11) 25% (12) Rs.100 per half year. (13) Rs.175 per half year
(14) Rs.1,00,000 (15) Rs. 23,606.25 (16) Rs. 32,142.86;
Rs. 17,857.14
(17) Rs.67,741.93; (18) Rs.15,853.65; (19) 8%; Rs. 1,000
Rs. 32,258.07 Rs.4,146.35
(20) Rs. 24,140 (21) Rs.3127.65 (22) Rs. 1,00,000;
Rs. 1,40,000
(23) 2 years and 6 months (24) Rs. 1,600

2.2 COMPOUND INTEREST

t the end of each interest period if the interest earned is added to the principal and thereafter

A earns interest, then the interest is said to be compounded. The sum of the original principal
and total interest is called the amount. Hence the difference between the amount and
original principal is called the compound interest. For example, let a person takes a loan of Rs.
10000 from a moneylender for two years at 20% per annum interest. At the end of the first year the
principal along with the interest will amount to
PNR
A1  P 
100
10000 1 20
i.e. A1  10000   12000.
100

10000 1 20
i.e. A1  10000   12000.
100

At the end of the second year, by using A1  12000 as the principal, it amounts to

A1 NR
A2  A1 
100

12000 1 20
i.e. A2  12000   14400
100

Hence the compounded interest is 14400 10000  4400 .

If P is the principal, R is the rate of interest, N is the time duration, the

the amount at the end of the year I :

PNR PR  R 
A1  P  P   P 1  .
100 100  100 

the amount at the end of the year II :

A1 NR AR  R 
A2  A1   A1  1  A1 1  .
100 100  100 

2
 R  R   R 
 A2  P 1  1    P 1  
 100  100   100 

Therefore the amount A at the end of N th year is given by:

N
 R 
A  AN  P1  
 100 

The rate interest is usually stated as an annual interest rate, referred to as the nominal rate of interest.

Solved Problems
1) Calculate the amount and compound interest on:

(i) Rs. 1,000 at 10 % p.a. for 5 years.

(ii) Rs. 12,000 at 5% p.a. for 7.5 years.


Solution:

(i) Given that P = 1000, R = 10% p.a. and N = 5 years


N 5
 R   10 
A  P 1    10001  
 100   100 
 1000(1.1) 5
 1000(1.61051)
 Rs.1610.51

(ii) Given that P = Rs.12,000, R = 5% p.a. and N = 7.5 years


N 7.5
 R   5 
A  P 1    120001  
 100   100 
 12000(1.05) 7.5
 12000(1.442)
 Rs.17304

2) Calculate the amount and compound interest on Rs. 8,000 for 2 years at 15% per annum
compounded half-yearly.

Solution:

We have P = Rs. 8,000, R = 15% p.a. = 7.5% per half year and

N = 2 years = 4 half years.


N

If A is the maturity amount, then A  P1 
R 
 .
 100 
4
 7.5 
Hence, A = 80001    8000 (1.15)  8000(1.3355)  Rs.10,684 .
2

 100 

Also the compound interest C.I. = A – P

= Rs. 10,684 – Rs. 8,000

= Rs. 2,684.

3) Calculate the amount and compound interest on Rs. 15,000 in 2 years, when the rates of interest
for successive years are 5% p.a and 6% p.a respectively.

Solution:
We have P = Rs. 15,000, N = 2 years .

During the first year R = R1 = 5% and during the second year R = R2 = 6%.

If A is the maturity amount, then A  P1  1 1  2  .


R R
 100  100 

Hence, A = 150001 
5  6 
1    15000 (1.05)(1.06)
 100  100 

 Rs.16,695.

Also the compound interest C.I. = A – P

= Rs. 16,695 – Rs. 15,000

= Rs. 1,695.

4) Divide Rs. 39,030 between A and B so that when their shares are lent out, the amount that A
receives in 2 years is the same as what B receives in 4 years. The interest is compounded
annually at the rate of 4% per annum.

Solution:

Let the share of A be Rs. x, and hence the share of B is Rs.(39030 - x)

Then from the given data, we have R =R1=R2= 4% and

Part I Part II

P1=Rs. x; N1 = 2 years P2 = Rs. (39030 - x); N2 = 4 years


N1 N2
 R 
A1 = P1 1  1 
 R 
A2 = P2 1  2 
 100   100 
2 4

= x 1 
4  
= (39030  x)1 
4 
 
 100   100 
2 4

Given that A1 = A2. Therefore, x1 
4   4 
 = (39030  x)1  
 100   100 

Simplifying the above expression, we get

x = 42214.85 = Rs. 20,280.


2.0816
5) Rs. 16,820 is divided between two persons Mr. X and Mr. Y, 27 and 25 years old respectively;
and their money is invested at 5% per annum compound interest in such a way that both receive
equal money at the age of 40 years. Find the share of each out of Rs. 16,820.

Solution:

Let the share of Mr. X be Rs. x, and hence the share of Mr.Y is Rs.(16820 - x)

Then from the given data, we have R = 5% and

Mr. X Mr. Y
P1=Rs. x; P2 = Rs. (16820 - x);
N1 = (40-27) years. N2 = (40-25) years
= 13 years. = 15 years
N1 N2
 R 
A1 = P1 1  1 
 R 
A2 = P2 1  2 
 100   100 
13 15

= x 1 
5  
= (16820  x)1 
5 
 
 100   100 
13 15

Given that A1 = A2. Therefore, x1 
5   5 
 = (16820  x)1  
 100   100 

Simplifying the above expression, we get

x = 18544.05 = Rs. 8,820.


2.1025

6) What principal invested today will amount to Rs.1,630.80 in 4 years at 13% per annum
compound interest?

Solution:

Given A = Rs. 1,630.80; N = 4 years and R = 13% p.a.


N 4
 R   13 
Since, A  P1   , we have 1630.80  P1  
 100   100 

1630.80
Hence P = = Rs. 1,000.
(1.13) 4

7) At what rate percent per annum will a sum of Rs. 50,000 become Rs. 80,000 if the loan given for
3 years attracts compound interest?

Solution:
Given P = Rs. 64,000; A = Rs. 1,25,000 and N = 3 years.
N 3
Since, A  P1 
R   R 
 , we have 125000  64,0001  
 100   100 

 1 
 125000  3 
Hence R =    1 100 = 25% p.a.
  64000 


8) In how many years will Rs. 12,000 amount to Rs. 21,148.11 at 12 % per annum compound
interest?

Solution:

Given P = Rs. 12,000; A = Rs. 21,148.11 and N = 3 years.


N N
 R   12 
Since, A  P1   , we have 21148.11  120001   .
 100   100 

Simplifying the above expression, we get

1.7623425 = (1.12)N ,

which implies (1.12)5 = (1.12)N.

Comparing, we get N = 5.

9) In how many years will a sum of money double itself if compound interest is applied at 5% per
annum?

Solution:

Let P be the principal. Given A = 2P and R = 5 % p.a.


N N
 R   5 
Since, A  P1   , we have 2 P  P1   .
 100   100 

Canceling P on both sides and simplifying, we get 2  (1.05) N ,

which implies (1.05)14.2 = (1.05)N .

Comparing, we get N = 14.2 years = 14 years and 2.4 months

10) Calculate the compound interest of Rs. 1,12,000 at 8% per annum for 2 years if the interest is
calculated half yearly. What difference will it make if interest is calculated quarterly.

Solution:

Given P = 1,12,000; R = 8% p.a. and N = 2 years.

Case I : ( When the interest is calculated half yearly )

Here R = 4 % per half year and N = 4 half years.


4
Maturity amount A1 = 112000 1 
4 

 100 

= Rs. 1,31,024.15

Case II : (When the interest is calculated quarterly )

Here R = 2% per quarter year and N = 8 quarter years.

8
Maturity amount A2 = 112000 1 
2 

 100 

= Rs. 1, 31, 225.85

Hence the difference between the compound interests that are calculated half yearly and quarterly
= A2 – A1 = Rs. 201.70

11) A loan of Rs. 2,25,000 to be repaid in two equal annual installments. If the rate of compound
interest is 13% per annum, find the installment amount.

Solution:

Given P = Rs. 2,2,5,000 and R = 13 % p.a.

Also given that the number of installments = 2

Let the equal installment amount = Rs. x.

Let A1 be the maturity amount at the end of the first period.

1
Therefore, A1 = 225000 1 
13 
 = 225000(1.13) = Rs. 2,54,250
 100 

If Rs. x is the first installment paid, then the principal left out for the next period is Rs. (254250 -
x).

If A2 is the maturity amount of Rs. (254250 – x) at the end of the second period, then A2 = Rs. x.
1
Hence, A2 = x = (254250 – x) 1 
13 
 .
 100 
1
Considering, x = (254250 – x) 1 
13 
 and simplifying, we get
 100 

x = 287302.50 = Rs. 1,34, 883.80


2.13

Therefore, the equal installment amount = Rs. x = Rs. 1,34, 883.80

12) A banker borrows a certain sum at 8% p.a. compound interest compounded half yearly. He lends
this money at 8% p.a. compound interest compounded quarterly. If he earns Rs. 9 in two years.
Find the sum borrowed.

Solution:

Let P be the principal. Given R = 8% p.a. and N = 2 years

Case I: When the interest is compounded half yearly.

Here R = 4% per half year and N = 4 half years.


4
Therefore the maturity amount A1 = P1 
4 
 = (1.169859) P
 100 

Case II: When the interest is compounded quarterly.

Here R = 2% per quarter year and N = 8 quarter years.


8
Therefore the maturity amount A2 = P1 
2 
 = (1.171659) P
 100 

Also given that the difference between the compound interests = A2 - A1= Rs. 9

But A2 - A1 = (1.171659) P - (1.169859) P

Therefore (1.171659) P - (1.169859) P = 9.

which on simplifying, we get P = Rs.5,000.

13) Find the difference between compound and simple interest on Rs. 15,000 invested for 6 years at
Rs. 13% per annum.

Solution:

Given P = Rs. 15,000, R = 13% p.a. and N = 6 years.


Simple interest = PNR = 15000  6  13 =Rs. 11,700
100 100

Compound interest = A – P
N
= P1 
R 
 P
 100 

 13 
6

= 150001    1 = Rs. 16,229.27
 100  

Difference between C.I and S.I = Rs. 16,229.27 - Rs. 11,700

= Rs. 4,529.27

Supplementary Problems
1) Calculate the amount and compound Interest on:
(a) Rs. 8,450 for 5 years at 5 % p.a.
(b) Rs. 9,990 for 2 years at 9 % p.a.
(c) Rs. 4,800 for 6 years at 10.9% p.a.
(d) Rs. 11,200 for 7 years at 11.2% p.a.
3
(e) Rs. 15,000 for 9 years at 6 % p.a.
7
(f) Rs. 1,40,000 for 5 years and 3 months at 4% p.a.
(g) Rs. 12,000 for 9 years and 6 months at 2.7% p. m.
(h) Rs. 14,300 for 2 years and 6 months at 1.9% p.m.
2
(i) Rs. 18,350 for 4 years and 7 months at 1 % p.m.
3
(j) Rs. 96,000 for 300 days at 7.3% p.a.
(k) Rs. 16,000 for 85 days at 4.7% p.a.
(l) Rs. 7,200 from 12th January to 27th March at 5% p.a.
(m) Rs. 1,58,000 from 6th October to 7th February at 12.5% p.a.
1
(n) Rs. 75,000 for 6th January to 7th July in a leap year at 2 % p.m.
4
(o) Rs. 8,600 for 279 days in 2004 at 5% p.a.
2) Calculate the amount and compound interest on Rs. 15,000 for 1 year at 12 % per annum
compounded half-yearly.
3) Calculate the amount and compound interest on Rs. 18,250 for 1 year at 8% per annum
compounded quarterly.
4) Calculate the amount and compound interest on Rs. 24,000 in 3 years, when the rates of interest
for successive years are 10%, 12%, and 15% respectively.
5) Calculate the amount and compound interest on Rs. 23,450 in 4 years, when the rates of interest
for successive years are 6% p.a., 8% p.a., 10% p.a. and 12% p.a. respectively.
6) Divide Rs. 54,000 between A and B so that when their shares are lent out, the amount that A
receives in 3 years is the same as what B receives in 6 years. The interest is compounded
annually at the rate of 6% per annum.
7) Divide Rs. 81,000 between A and B so that when their shares are lent out, the amount that A
receives in 5 years is the same as what B receives in 10 years. The interest is compounded
annually at the rate of 8% per annum.
8) What principal invested today will amount to Rs.22,039.92 in 5 years at 8% per annum
compound interest?
9) What principal invested today will amount to Rs. 33,788.47 in 10 years at 6.5% per annum
compound interest?
10) At what rate percent per annum will a sum of Rs. 15,000 become Rs. 25,000 if the loan given for
5 years attracts compound interest?
11) At what rate percent per annum will a sum of Rs. 85,000 doubles itself if the investment for 7
years attracts compound interest?
12) If the compound interest on certain sum after 2 years is Rs. 6741 and Rs. 212.87, after 3 years.
What is the rate of interest?
13) In how many years will Rs. 21,000 amount to Rs. 28,142.01 at 5 % per annum compound
interest?
14) In how many years will Rs. 16,500 amount to Rs. 19,332.37 at 2% per annum compound interest.
15) In what time a sum will double itself at 13% per annum compounded interest, when the interest is
computed half yearly.
16) A sum is lent on compound interest at 12% p.a. calculated quarterly is doubled in a given time.
Find the time period.
17) A sum invested at a certain rate of compound interest doubles itself in 6 years. In how many
years will it become eight times of the sum?
18) The simple interest on a certain sum at certain rate is Rs. 480 for 2 years and the compound
interest on the same sum and at same rate for 2 years is Rs. 508.80. Find the sum and the rate of
interest.
19) Find the difference between the compound interests compounded yearly and half-yearly on Rs.
4,00,000 for 2 years at 10% rate of interest per annum.
20) Calculate the compound interest of Rs. 52,000 at 11% per annum for 3 years if the interest is
calculated half yearly. What difference will it make if interest is calculated quarterly.
21) A loan of Rs. 10,200 to be repaid in two equal annual installments. If the rate of compound
interest is 7.5% per annum, find the installment amount.
22) A loan of Rs. 1,00,000 to be repaid in two equal annual installments. If the rate of compound
interest is 12.5% per annum, find the installment amount.
23) A person takes a loan from a moneylender. He pays back the loan in two equal annual
installments of Rs. 982. If the rate of compound interest is 6% per annum, find the loan.
24) A person takes a loan from a moneylender. He pays back the loan in two equal annual
installments of Rs. 1677.69. If the rate of compound interest is 7.8% per annum, find the loan.
25) A banker borrows a certain sum at 11% p.a. compound interest compounded annually. He lends
this money at 11% p.a. compound interest compounded quarterly. If he earns Rs. 46.26 in two
years, find the sum borrowed.
2
26) A banker borrows a certain sum at 8 % per annum compound interest compounded half yearly.
3
He lends this money at 9 % per annum compound interest compounded quarterly. If he earns Rs.
212.08 fin four years, find the sum borrowed.
27) A banker borrows a certain sum at 10% per annum compound interest compounded annually. He
lends this money at 12% per annum compound interest compounded quarterly. If he earns Rs.
17,544.85 in three years, find the sum borrowed.
28) Find the difference between the compound interest ant simple interest of Rs.12,000 for 4.5 years
at the interest rate of 7% per annum.
29) On what sum the difference between its compound interest and simple interest of 2 years at 6%
per annum is Rs.39.60.
30) A person borrowed Rs. 22,400 at 10% per annum simple interest and invested at 9% per annum
compound interest, compounded half yearly. If the transaction was for 3 years, what was his
gain.

Answers:

(1) (a) Rs. 10,782.20; (1) (b) Rs. 11,869.12; (1) (c) Rs. 8,929.44;

Rs.3,332.20 Rs. 1,879.12 Rs. 4129.44

(1) (d) Rs. 23,548 (1) (e) Rs. 26,283. (1) (f) Rs. 1,72,009.74

Rs. 12,348 Rs. 11,283. Rs. 32,009.74

(1) (g) Rs. 172630.33 (1) (h) Rs. 23,896.37 (1) (i) Rs. 42,317.86

Rs. 160630.33 Rs. 9,596.37 Rs. 23,967.86

(1) (j) Rs. 1,01,723.59 (1) (k) Rs. 16,172.05 (1) (l) Rs. 7,271.58

Rs. 5,723.59 Rs. 172.05 Rs. 71.58

(1) (m) Rs. 1,64,981.92 (1) (n) Rs. 84,520.70 (1) (o) Rs. 8,925.87

Rs. 6,981.92 Rs. 9,520.70 Rs. 325.87

(2) Rs. 16,854; Rs. 1,854 (3) Rs. 19,754, Rs. 1,504 (4) Rs. 34003.20; Rs. 10003.20

(5) Rs. 33,073.73; Rs. 9,623,73 (6) Rs. 29,353.90; Rs.20,646.10 (7) Rs. 48197.58; Rs. 32,802.42

(8) Rs. 15,000 (9) Rs. 18,000 (10) 10.8% p.a.

(11) 10.4% p.a. (12) 9 % p.a. (13) 6 years.

(14) 8 years. (15) 11 years (16) 7 years

(17) 18 years (18) Rs. 2,000; 12% p.a. (19) Rs. 2,202.50

(20) Rs. 308.93 (21) Rs. 5,680.66 (22) Rs. 59,558.82

(23) Rs. 1798.61 (24) Rs. 3,000 (25) Rs. 4,500

(26) Rs. 9,000 (27) Rs. 25,000 (28) Rs. 490.77

(29) Rs. 11,000 (30) Rs. Rs. 50.62


2.2.1 Depreciation:

Every item purchased are subject to decrease in its value as the time increases, due to its
utility. This decrease in its value is termed as depreciation. To calculate the depreciated value by
declining balance method we use the following formula:
N
 R 
A  P1  
 100 

where P = the value of the item at the time of purchase,

R = rate of depreciation per annum,

N = life of the item in years,

A = the depreciated value.

Solved Problems
1) An instrument costs Rs. 2,00,000. If the rate of compound depreciation is 13.5% per annum.
Find the depreciation cost of the instrument cost after 8 years.

Solution:

Given P = Rs. 2,00,000; N = 8 years and R = 13.5 % p.a.


N
Since the depreciated value A  P1 
R 
 ,
 100 
8
we have A  2000001 
13.5  8
 = 200000 (0.865) .
 100 

Hence the depreciated value A = 200000 (0.3134) = Rs. 62,860.

2) The cost of the scooter is Rs.52,000 and its expected life is 10 years. If the depreciated value of
the scooter is Rs. 12,000, calculate the rate of depreciation per year.

Solution:

Given that P = 52,000, A = 12,000 and N = 10 years.


N
The depreciated value A  P1 
R 
 .
 100 
10
Substituting the given values, we get 12000  520001 
R 

 100 
10
which implies, 1 
R  12000 10
   0.2308 =(0.8636) ,
 100  52000

comparing, we get 1 


R 
  0.8636 .
 100 

Therefore, R  13.64% p.a.

3) The initial cost of the appliance is Rs. 2,32,000. If the cost of the appliance is depreciated at the
end of each year by 10%, in how many years its depreciated value will become Rs.28,205.87.

Solution:

Given that P = 2,32,000, A = 28,205.87 and R = 10 % p.a.


N
The depreciated value A  P1 
R 
 .
 100 
N
Substituting the given values, we get 28205.87  2320001 
10 
 .
 100 

Simplifying, we have 0.9  0.121577  0.9 , and


N 20

comparing, we get N = 20 years.

4) The book value of the asset at the end of the 4th year is Rs. 10,00,000 and at the end of the 10th
year is Rs. 5,31,441. If depreciation is written off under the declining balance method, find the
rate of depreciation per annum and the original cost of the asset.

Solution:

Let the original cost of the asset be Rs. x and let the rate of depreciation per annum be R.
4
Book value at the end of 4th year is 10,00,000  x1 
R 
 ---(1)
 100 
10
Book value at the end of 10 year is 5,31,441  x1 
th R 
 ----(2)
 100 
10
 R 
x 1  
Dividing Equation (2) by equation (1), we get
5,31, 441
  100 
10, 00, 000 4
 R 
x 1  
 100 
6 6
 R  5,31,441  9 
Simplifying, we get 1       , and
 100  10,00,000  10 

 R  9
comparing, we get 1      , which implies, R = 10 %
 100   10 

5) A manufacturer estimates that the value of his machinery depreciates by 13% of its value at the
beginning of the year. Find the original value of the machine, if it depreciates by Rs. 5,655
during the second year.

Solution:

Given than R = 13 % p.a. and let the initial value of the machine = Rs. P

Then the depreciated value at the end of first year, A1 = P1 


13 
 = Rs. 0.87P
 100 

Also the depreciated value at the end of second year, A2 = 0.87 P1 
13 

 100 

= Rs. 0.7569 P.

Hence the depreciation during the second year = A1 – A2

= 0.87 P – Rs. 0.7569 P

= 0.1131 P = Rs. 5655 (given)

Therefore, P = 5655 = Rs. 50,000.


0.1131

6) A machine was depreciated at the rate of 15% per annum for the first 2 years after it was
purchased and at the rate of 10% for the next three years. The original cost of the asset was Rs.
20,000. Find the depreciated value of the asset at the end of five years.

Solution:

Given that the Original Cost of the asset, P = Rs. 20,000

The rate of depreciation for the first two years = R1 = 15% p.a.
The rate of depreciation for the next three years = R2 = 10 % p.a.
2 3
 R   R 
Therefore, the depreciated value A = P1  1  1  2 
 100   100 
2 3
= 200001 
15   10 
 1   = Rs. 10,534.05.
 100   100 

Supplementary Problems

1) An instrument costs Rs. 58,000. If the rate of compound depreciation is 10% per annum. Find
the depreciation cost of the instrument after 5 years.

2) An instrument costs Rs. 3,25,000. If the rate of compound depreciation is 15.2% per annum.
Find the depreciation cost of the instrument after 15 years.

3) An instrument costs Rs. 12,00,000. If the rate of compound depreciation is 9% per annum. Find
the depreciation cost of the instrument after 12 years.

4) A Company purchased a concrete mixing machine for Rs. 15,00,000. The machine has useful
life of 6 years. If the company writes off depreciation at the rate of 30% p.a. under the declining
balance method, compute the terminal book value of the machine.

5) The cost of a car is Rs. 4,00,000 and its expected life is 5 years. If the depreciated value of the
car is Rs. 1,50,000, calculate the rate of depreciation per year.

6) The cost of the vehicle is Rs. 4,00,000 and its expected life is 7 years. If the depreciated value of
the car is Rs. 2,00,000, calculate the rate of depreciation per year.

7) The initial cost of the vehicle is Rs. 7,50,000. If the cost of the appliance is depreciated at the
end of each year by 15%, in how many years its depreciated value will become Rs.65,515.50

8) A person purchased a car at a cost of Rs. 12,00,000. After few years he sold the car for Rs.
9,00,000 at depreciation rate of 12% per annum. In how many years after purchase he has sold
his car.

9) A machine purchased at Rs.32,00,000 is being depreciated under the declining balance method at
the rate of 15% per annum. In how many years its depreciated value will become Rs.14,19,857.

10) A machine is depreciated at the rate of 20% on declining balance method. The original cost of
the machine is Rs. 1,00,000 and its scrap value was estimated at Rs. 30,000. What is the
productive life of the machine?

11) The book value of the asset at the end of the 7th year is Rs. 62,50,000 and at the end of the 11th
year is Rs. 44,77,456. If depreciation is written off under the declining balance method, find the
rate of depreciation per annum and the original cost of the asset.
12) A manufacturer estimates that the value of his machinery depreciates by 10% of its value at the
beginning of the year. Find the original value of the machine, if it depreciates by Rs. 9,000
during the second year

13) A manufacturer estimates that the value of his machinery depreciates by 22% of its value at the
beginning of the year. Find the original value of the machine, if it depreciates by Rs. 9,438
during the second year

14) A manufacturer estimates that the value of his machinery depreciates by 17% of its value at the
beginning of the year. Find the original value of the machine, if it depreciates by Rs. 8,783.48
during the third year.

15) A machine was depreciated at the rate of 12% per annum for the first 2 years after it was
purchased and at the rate of 16% for the next three years. The original cost of the asset was Rs.
39,000. Find the depreciated value of the asset at the end of five years

16) A machine was depreciated at the rate of 18% per annum for the first 3 years after it was
purchased and at the rate of 20% for the next three years. The original cost of the asset was Rs.
13,00,000. Find the depreciated value of the asset at the end of six years

17) A machine was depreciated at the rate of 10% per annum for the first 2 years after it was
purchased, at the rate of 12% per annum for the next three years and at the rate of 15% per
annum for the next five years. The original cost of the asset was Rs.58,00,000. Find the
depreciated value of the asset at the end of ten years

18) A machine was depreciated at the rate of 20% per annum for the first 2 years after it was
purchased, at the rate of 25% per annum for the next two years and at the rate of 30% per annum
for the next one year. The original cost of the asset was Rs.25,50,000. Find the depreciated
value of the asset at the end of ten years

Answers

(1) Rs. 32,248.42 (2) Rs. 27,404.42 (3) Rs. 3,86,971.20

(4) Rs. 1,76,473.50 (5) 17.81% p.a. (6) 9.5% p.a.

(7) 15 years (8) 2.25 years (9) 5 years

(10) 5.5 years approx. (11) 8 % p.a. (12) Rs. 1,00,000

(13) Rs. 55,000 (14) Rs. 75,000 (15) Rs. 16,983

(16) Rs. 3,66,990.54 (17) Rs. 14,20,547.60 (18) Rs. 6,42,600

2.2.2 Nominal Rate and Effective Rate:

Nominal rate of interest is the rate of interest received when the conversion period is one
year. When the conversion period is less than one year, the realized rate of interest is the effective
rate of interest, which will be greater than the Nominal rate. The effective rate of interest is
computed as follows:
q
 i
r  1    1
 q

R
where i  , R = the rate of interest, r = effective rate of interest, q = number of times interest is
100
computed in a year.

Solved Problems.

1) Find the nominal and effective rates of interest in each of the following cases:

(i) Rs. 10,000 lent at 10% p.a., interest payable half yearly.

(ii) Rs. 7,500 lent at 12% p.a. interest payable quarterly.

(iii) Rs. 40,000 invested at 16% p.a. interest payable monthly.

(iv) Rs. 1,00,000 lent at 8 % p.a. interest payable annually.

Solution:

10
(i) Nominal rate is the given rate per annum = 10%. (i.e.) given R = 10%, implies i   0.1 .
100
Also given that the number of times interest computed in a year is q = 2.

Hence the effective rate of interest


q
 i 2
r  1    1  1 
0.10 
 1  1.05  1
2

 2 
 q

= 1.1025  1 = 10.25 %

(ii) Nominal rate is the given rate per annum = 12%. (i.e.) given R = 12%, implies

12
i  0.12 . Also given that the number of times interest computed in a year is q = 4.
100

Hence the effective rate of interest


q
 i
4
 1  1 
0.12 
r  1   1  1.03  1
4
  
 q  4 

= 1.1255  1 = 0.1255 = 12.6 %


(iii) Nominal rate is the given rate per annum = 16%. (i.e.) given R = 16%, implies
16
i  0.16 . Also given that the number of times interest computed in a year is q = 12.
100

Hence the effective rate of interest


q
 i
12
 1  1 
0.16 
r  1   1  1.01333  1
12
  
 q  12 

= 1.172  1

= 0.172 = 17.2 %

8
(iv) Nominal rate is the given rate per annum = 8%. (i.e.) given R = 8% implies i   0.08 .
100
Also given that the number of times interest computed in a year is q = 1.

Therefore the effective rate of interest = nominal rate.

We shall verify it.


q
 i
1
 1  1 
0.08 
The effective rate of interest r  1   1
 q 
 1 

 1.08  1

= 0.08 = 8 %.

2) Find the amount of Rs. 3,000 lent for 10 years at 15% per annum compound interest payable
quarterly.

Solution:

Method I :

By the usual method, we can compute the compound interest as follows:

Given P = Rs. 3,000; N = 10 years = 40 quarters

15 15
R = 15% p.a. = % p. q., implies i  = 0.15 ( per annum )
4 100

N 40
 R   15 
The maturity amount A  P1   = 3,0001  
 100   400 

= Rs. 3000  4.36


= Rs. 13,080.

Hence the compound interest C.I. = A – P = Rs. 13,080 – Rs. 3,000

= Rs.10,080.

Method – II:

We can also compute the compound interest by using the effective rate and is done

as follows:
q
 i
4
 1  1 
0.15 
The effective rate r  1  1
 q   4 

=0.1587 = 15.87 %.

N
Since, A  P1 
R 
 , substituting we get
 100 
10
 15.87 
A  3,0001  
 100 

= 3,0001  0.1587
10

=3,000(4.36) = Rs. 13,080.

Hence the compound interest C.I. = A – P = Rs. 13,080 – Rs. 3,000

= Rs.10,080.

3) In what time will a sum of Rs.12,000 becomes Rs.18,000 at 10% per annum compound interest
payable half yearly?

Solution:
q
 i
2
 1  1 
0.1
The effective rate r  1   1
 q 
 2 

= 1.1025 – 1= 0.1025

= 10.25%
N N
Substituting in A  P1 
r   10.25  .
 , we get 18,000  12,0001  
 100   100 
1.5 = 1.1025 , which can be written as
N
Simplifying, we get

1.5 = (1.1025) 4.16  1.1025 .


N

Hence by comparing N = 4.16 years.

4) Find the present value of an amount of


Rs. 2,400 due 5 years form now, if the compound interest rate is 10%, reckoned half yearly.

Solution:
q
 i
2
 1  1 
0.10 
The effective rate r  1   1
 q 
 2 

= 1.1025 – 1= 0.1025 = 10.25%


N 5
Substituting in A  P1 
r   10.25  .
 , we get 2,400  P1  
 100   100 

2,400
Simplifying, we get P  = Rs.2176.87.
1.10255
5) A person wishes to have Rs.25,000 in cash 5 years hence. He has decided to open a fixed deposit
account with a bank that pays 8% per annum as interest. If the bank compounds interest once in
six months, how much should he deposit in the bank today?

Solution:
q
 i
2
 1  1 
0.08 
The effective rate r  1    1 = 1.0816 – 1
 q 
 2 

= 0.0816 = 8.16 %.
N 5
Substituting in A  P1 
r   8.16  .
 , we get 25,000  P1  
 100   100 

25,000
Simplifying, we get P  = Rs.16891.89.
1.08165
Supplementary Problems
1) Find the effective rate of interest when a sum lent at 20% per annum is compounded half yearly.
1
2) Find the effective rate of interest when a sum lent at 15 2 % per annum is compounded half
yearly.
3) Find the effective rate of interest when a sum lent at 12% per annum is compounded quarterly.
4) Find the effective rate of interest when a sum lent at 17.35% per annum is compounded quarterly.
5) Find the effective rate of interest when a sum lent at 18% per annum is compounded monthly.
6) Find the effective rate of interest when a sum lent at 13.75% per annum is compounded monthly.
7) Find the amount of Rs. 15,000 lent out for 6 years at 8% per annum compound interest payable
quarterly.
8) When a child was born, a sum of Rs. 20,000 was placed in a bank account for the child’s benefit.
The bank pays 12% per annum interest compounded half yearly. What amount of money the
child would receive on its fifth birthday?
9) Suppose you deposit Rs. 10000 with a bank, which pays 16% interest with quarterly
compounding. How much will this deposit grow in 5 years?
10) How much would a deposit of Rs. 8000 at the end of 3 years be, if the nominal rate is 12% and if
the compounding is done quarterly?
11) What is the difference between the effective rate of interest and nominal rate of interest in the
following cases:
Case A: Nominal rate of interest is 15% and the frequency of compounding is twice a year.
Case B: Nominal rate of interest is 24% and the frequency of compounding is six times a
year.
Case C: Nominal rate of interest is 18% and the frequency of compounding is twelve times a
year.
12) A banker borrows money at 12.5% per annum reckoning interest annually and lends the same to
his customer at 12.5% per annum reckoning interest semi-annually. He gains Rs. 585 in the first
year on this transaction. What was the sum borrowed and lent?

Answers

(1) 21% (2) 16.1% (3) 12.55%

(4) 18.51% (5) 19.56% (6) 14.65%

(7) Rs. 24,126.55 (8) Rs. 35,816.95 (9) Rs. 21911.23

(10) Rs. 11406.09 (11) 15.56%, 26.53 %, 19.56% (12) Rs. 1,50,000

2.3 ANNUITY
An annuity is a sequence of payments, usually equal, during the life of the annuitant ( the receiver )
in return for the investment of a lump sum. For example, if a person deposits a lump sum amount of
Rs. 10000/- in a financial company in turn agrees to pay Rs. 2000 annually after a lapse of 6 years
throughout the life of the depositor. The depositor is called annuitant, and the annual payment
received by him is called annuity. Premiums on insurance, mortgage payments, interest payments on
bonds, payments of rent, payment on installment purchases, and dividends are all annuities.
The time between successive payments of an annuity is called the payment interval. The time from
the beginning of the first payment interval to the end of the last payment interval is called the term of
an annuity.
Annuity Certain

When the term of an annuity is fixed, i.e. the dates of the first and last payments are fixed, the
annuity is called an annuity certain.

Annuity Contingent:

When the term of the annuity depends on some uncertain event, the annuity is called a contingent
annuity.

Note: Unless otherwise specified, the word "annuity" will refer to an annuity certain.

Annuity immediate:

When the payments are made at the end of the payment intervals, the annuity is called an ordinary
annuity or annuity immediate.

Annuity Due:

When the payments are made at the beginning of the payment intervals, the annuity is called an
annuity due.

Deferred annuity:

A deferred annuity is an annuity whose first payment is due at some later date.

Perpetuities:

Perpetuity is an annuity whose payments begin on a fixed date and continue forever.

When the payment interval and interest conversion period coincide, the annuity is called a simple
annuity; otherwise, it is a general annuity.

We define the accumulated value of an annuity as the equivalent dated value of the set of payments
due, at the end of the term. Similarly, the discounted value of an annuity is defined as the equivalent
dated value of the set of payments due, at the beginning of the term.

We shall use the following notation:

A = periodic payment of the annuity.

N = number of interest conversion periods during the term of an annuity

R = interval rate per conversion period

V = accumulated value, or the amount of the an annuity

P = discounted value, or the present value, of an annuity.


2.3.1 ACCUMULATED VALUE OF AN ANNUITY
IMMEDIATE (ORDINARY SIMPLE ANNUITY):

Let A be the ordinary annuity and R% be the rate of interest per period. In ordinary annuity
the first installment is paid at the end of each period (FIGURE 2.1) and hence it earns interest for (N-1)
period, second installment is paid at the end of second period and hence earns interest for (N-2)
periods, third installment is paid at the end of third period and hence earns interest for (N-3) periods
and so on. Note that the last installment is paid at the end of the last duration and hence it can earn
interest for (N-N) periods only. (i.e. it earns no interest).

0 1 2 …. …. …. …. …. N-1 N

FIGURE 2.1

 if i  R then
100

the maturity amount of 1st annuity = A(1  i) ( N 1)

the maturity amount of 2nd annuity = A(1  i) ( N 2)

the maturity amount of 3rd annuity = A(1  i) ( N 3)

the maturity amount of (N-1)th annuity = A(1  i)1

the maturity amount of Nth annuity = A(1  i) 0

Thus summing up the above maturity amounts obtained for the sequence of payments we get the
accumulated value of the ordinary annuity.

(i.e.) accumulated value V = A(1  i) ( N 1) + A(1  i) ( N 2)

+ A(1  i) ( N 3) + + A(1  i)1 + A(1  i) 0


= A (1  i) ( N 1)  (1  i) ( N 2)    (1  i)  1
 (1  i ) N  1  A
= A  = (1  i) N  1 .
 i  i
2.3.2 PRESENT VALUE OF AN ANNUITY IMMEDIATE
(ORDINARY SIMPLE ANNUITY):

Present value of an ordinary simple annuity is the amount of money that must be invested
now (i.e. at the beginning of the term) at a certain rate of interest so that the payments due in future
can be obtained.

Let Rs. A be the annuity paid at R% for N years, which results in the accumulated value of
Rs.V. If Rs. P is the lump sum amount, which can yield the Rs. V, when invested at the time of the
first payment of the annuity, invested for N years at the interest rate of R% per year. then Rs. P is the
Present Value of the immediate annuity A.

Hence V  P1  i   P  V 1  i 
N N
(2.3.2a)

Since V is the accumulated value obtained using V 


A
i
 
(1  i) N  1 , using (2.3.2a) we get,

P
A
i
 
(1  i) N  1 1  i 
N

Simplifying, we have P 
A
i

1  1  i  
N

A  (1  i) N  1 

i  (1  i) N 
.

Equivalently P is the accumulated value of the present worth of every annuity paid. Hence, for
annuity immediate, we have

P  A1  i   A1  i   A1  i     A(1  i)  N


1 2 3

A A A A
(i.e.) P       (2.3.2b)
(1  i) (1  i) 2
(1  i) 3
(1  i) N

Multiplying throughout by (1  i) , we get

A A A
P(1  i)  A     
(1  i) 1
(1  i) 2
(1  i) N 1 (2.3.2c)

A
Hence (2.3.2b) – (2.3.2c), gives P(1  i)  P  A  , which on simplifying gives
(1  i) N

 1 
P(i)  A1  
 (1  i)
N

A  (1  i) N  1  A
i  (1  i) N  i 1  (1  i) 
Therefore, the present value P   = N .
Solved Problems
1) Find the amount of an ordinary annuity of Rs. 3000 for 12 years at the rate of interest of 5% per
annum.

Solution:

5
Given A = Rs. 3000; N = 12 years;R = 5% p.a., implies i  = 0.05.
100

Therefore, the accumulated value V 


A
i
 
(1  i) N  1

=
3000
0.05

(1  0.05)12  1 
= Rs. 47,751.40.

2) Find the amount of an ordinary annuity of 15 monthly payments of Rs. 2500 that earn interest at
12% per annum compounded monthly.

Solution:

1
Given A = Rs. 2500; N = 15 months;R = 12% p.a. =1% p.m, implies i  = 0.01
100

Therefore, the accumulated Value V 


A
i

(1  i) N  1 

=
2500
0.01

(1  0.01)15  1 
= Rs. 40,242.24.

3) A man deposits Rs. 30000 at the end of each year for 20 years. He made his first payment at the
end of year 1991 and last payment at the end of year 2011. How mush should be there in his
account on 31 Dec. 2011, if the 10% interest rate is compounded annually.

Solution:

10
Given A = Rs. 3000; N = 12 years; R = 10% p.a., implies i  = 0.1
100

Therefore, the accumulated value V 


A
i
 
(1  i) N  1
=
30000
0.1

(1  0.1) 20  1 
= Rs. 17,18,249.99.

4) A person is repaying a debt with payments of Rs.2,500 per month. If he misses his payments for
July, August, September and October, what payment will be required in November to put him
back on schedule, if interest is at 12.6% per annum?

Solution:

The person has to pay four overdue payments with interest, along with November payment.
Considering only these five payments, we can find the accumulated value to be paid in
November, so that he is put back on schedule.

Given A = Rs. 2,500; N = 5 months; R = 12.5% p.a. =1.05% p.m.,

1.05
From R we get i  = 0.0105
100

Therefore, the accumulated Value V 


A
i

(1  i) N  1 
=
2,500
0.0105
 
(1  0.0105) 5  1 = Rs. 12,765.27.

5) What amount should be set-aside at the end of each year to amount to Rs.38,674.58 at the end of
10 years at 14% per annum compounded annually?

Solution:

14
Given V = Rs. 38,674.58; N = 10 years; R = 14% p.a., implies i  = 0.14.
100
Therefore, the accumulated value V 
A
i

(1  i) N  1 . 
Substituting, the values from the given data, we get

38674.58 
A
0.14

(1  0.14)10  1 
Simplifying the above expression, we get A = Rs. 2000.

6) A bank pays 8% per year interest, compounded quarterly. What equal deposits have to be made
at the end of each quarter for 10 years, to have Rs. 30,200 at the end of 10 years?

Solution:

Given V = Rs. 30,200; N = 10 years = 40 quarter years


8 2
R = 8 % p.a.= per quarter year = 2 p.q.y, implies i  = 0.02
4 100

Therefore, the accumulated value V 


A
i

(1  i) N  1 
Substituting, the values from the given data, we get

30,200 
A
0.02

(1  0.02) 40  1 
,

Simplifying, we get A = Rs. 500.

7) A machine costs Rs. 32,00,000 and its effective life is estimated 15 years. If the scrap value is
Rs.2,00,000. What amount should be retained out of profits at the end of each year to accumulate
compound interest at 8.5% per annum compounded annually?

Solution:

Given that, Cost of the machine = Rs. 32,00,000, Scrap value = Rs. 2,00,000

The amount desired at the end of 20 years = Rs. 32,00,000 – 2,00,000


= Rs. 30,00,000

Therefore, V = Rs. 30,00,000;N = 15 years; R = 8.5% which

8.5
implies i  =0.085.
100

We know that, the accumulated value V 


A
i
 
(1  i) N  1 .

which implies, 30,00,000 


A
0.085

(1  0.085)15  1 . 
Simplifying, we get A = Rs. 1,06,250.

8) Find the present value of an ordinary annuity of Rs. 1000 per annum for 10 years allowing
compound interest at 5% per annum.

Solution:

5
Given A= Rs. 1,000; N = 10 years; R = 5 % p.a. which implies i  = 0.05
100

Therefore, the present value P 


A
i

1  1  i 
N
 
1000
0.05

1  1  0.05
10

= Rs. 7,722.
9) Find the present worth of an ordinary annuity of Rs. 500 payable at the end of each month for 2
years, if the money is worth 8% compounded monthly.

Solution:

Given A = Rs. 500; N = 2 years = (2  12) years = 24 months

8 2
R = 8 % p.a.= % p.m.= % p.m.
12 3

2
i =0.00667
300

Therefore, the present value P 


A
i

1  1  i 
N

=
500
0.00667

1  1  0.00667
 24

= Rs. 31,484.26

10) Find the present value of an annuity of Rs. 900 payable at the end of 6 months for 5 years if the
money worth 8% is compounded semi annually.

Solution:

Given A = Rs. 900; N = 5 years = (5  2) half years = 10 h.y,

8 4
R = 8 % p.a.= % p.h.y. = 4% p.h.y., implies i  = 0.04
2 100

Therefore, the present value P 


A
i

1  1  i 
N

=
900
0.04

1  1  0.04
10

= Rs. 7,299

11) Find the present value and amount of an annuity of Rs. 500 payable once in two months for 2
years, if the money is worth 10% compounded once in two months.

Solution:

Given A = Rs. 500; N = 2 years = (2  6) two month periods = 12 t.m.p


10 5
R = 10 % p.a.= % per t.m.p., implies i  = 0.016667
6 300

Therefore, the present value P 


A
i

1  1  i 
N

=
500
0.016667

1  1  0.016667
12

= Rs. 5,400.

Supplementary Problems

1) Find the amount of an ordinary annuity of Rs. 5750 per annum for 10 years at the rate of interest
of 10% per period.

2) Find the amount of an ordinary annuity of Rs. 10000 per annum for 20 years at the rate of interest
of 8% per period.

3) Find the amount of an ordinary annuity of Rs. 7600 per annum for 15 years at the rate of interest
of 12% per period.

4) Find the amount of an ordinary annuity of 36 monthly payments of Rs. 12,000 that earn interest
at 12% per annum compounded monthly.

5) Find the amount of an ordinary annuity of 24 monthly payments of Rs. 5000 that earn interest at
6% per annum compounded monthly.

6) Find the amount of an ordinary annuity of 18 monthly payments of Rs. 8,500 that earn interest at
14.5% per annum compounded monthly.

7) A person is repaying a debt with payments of Rs.1,000 per month. If he misses his payments for
January, February, March, April and May, what payment will be required in June to put him back
on schedule, if interest is at 12% per annum?

8) A person is repaying a debt with payments of Rs.1,650 per month. If he misses his payments for
July, August, September, October and November, what payment will be required in December to
put him back on schedule, if interest is at 8.5% per annum?

9) What amount should be set-aside at the end of each year to amount to Rs. 23,466.40 at the end of
5 years at 8% per annum compounded annually?

10) What amount should be set-aside at the end of each year to amount to Rs. 32,647.96 at the end of
15 years at 7.5% per annum compounded annually?

11) A bank pays 15% per year interest, compounded quarterly. What equal deposits have to be made
at the end of each quarter for 12 years, to have Rs. 1,00,000 at the end of 12 years?
12) A bank pays 12.5% per year interest, compounded half-yearly. What equal deposits have to be
made at the end of each quarter for 20 years, to have Rs.3,00,000 at the end of 20 years?

13) A machine costs Rs. 66,00,000 and its effective life is estimated 10 years. If the scrap value is
Rs.1,00,000. What amount should be retained out of profits at the end of each year to accumulate
at compound interest at 12% per annum compounded annually?

Answers:

1) Rs. 91640.19 2) Rs. 4,57,619.64 3) Rs. 2,83,325.83

4) Rs. 5,16,922.54 5) Rs. 1,27,200.00 6) Rs. 1,69,830.00

7) Rs.6152.02 8) Rs. 10,000.97 9) Rs.4000.00

10) Rs.1250.00 11) Rs. 772.56 12) Rs.1820.39

13) Rs. 3,69,668.25

2.3.3 ACCUMULATED VALUE OF AN ANNUITY DUE:

Let A be the ordinary annuity and R% be the rate of interest per period. In ordinary annuity
the first installment is paid at the beginning of each period (FIGURE 2.3.3 a) and hence it earns interest for
N periods, second installment is paid at the end of second period and hence earns interest for (N-1)
periods, third installment is paid at the end of third period and hence earns interest for (N-2) periods
and so on. Note that the last installment is paid at the beginning of the last duration and hence it can
earn interest for one period only.

0 1 2 …. …. …. …. …. N-1 N

FIGURE 2.3.3 a

 if i  R then
100

the maturity amount of 1st annuity = A(1  i) N

the maturity amount of 2nd annuity = A(1  i) ( N 1)

the maturity amount of 3rd annuity = A(1  i) ( N 2)


the maturity amount of (N-1)th annuity = A(1  i) 2

the maturity amount of Nth annuity = A(1  i)1

Thus summing up the above maturity amounts, obtained for the sequence of payments, we get the
accumulated value of the ordinary annuity.

(i.e.) accumulated value V = A(1  i) N + A(1  i) ( N 1)

+ A(1  i) ( N 2) + + A(1  i) 2 + A(1  i)1

 
= A (1  i) ( N 1)  (1  i) ( N 2)    (1  i)  1 (1  i)

 (1  i ) N  1 
= 
A (1  i )
 i 

=
A
i
 
(1  i) N  1 (1  i) .

2.3.4 PRESENT VALUE OF AN ANNUITY DUE:


We know that the present value P is the accumulated value of the present worth of every
annuity paid. Hence incase of annuity due the present value P is given by

 P  A1  i 0  A1  i 1  A1  i 2    A(1  i) ( N 1)

A A A
 P  A     (2.3.4a)
(1  i) (1  i) 2
(1  i) N 1

P A A A A
       (2.3.4b)
(1  i) (1  i) 1
(1  i) 2
(1  i) 3
(1  i) N

Hence (2.3.4a) – (2.3.4b)

P A
 P  A
(1  i) (1  i) N

 i   1 
 P 
1 i 
A1 
 (1  i)
N

 
 = A 1  (1  i)  N 1  i 

or

A  (1  i) N  1 
P  (1  i)
i  (1  i) N 
.
Solved Problems
1) Find the amount of an annuity of Rs. 3000 per annum payable at the beginning of each year for
12 years at the rate of interest of 5% per period.

Solution:

Since the periodic payment is made at the beginning of each period, the annuity is the
annuity due. Given

5
A = Rs. 3000; N = 12 years; R = 5% p.a., implies i  = 0.05
100

Therefore, the accumulated Value V 


A
i
 
(1  i) N  1 1  i 

=
3000
0.05
 
(1  0.05)12  1 1  0.05

= Rs. 50,139

2) Find the amount of an annuity of 15 monthly payments of Rs. 2500, made at the beginning of
each period, that earn interest at 12% per annum compounded monthly.

Solution:

Given A = Rs. 2500; N = 15 months; R = 12% p.a. =1% p.m.

1
which implies i  = 0.01
100

Therefore, the accumulated Value V 


A
i
 
(1  i) N  1 1  i 

=
2500
0.01
 
(1  0.01)15  1 (1  0.01)

= Rs. 40,644.66

3) A man deposits Rs. 30000 at the beginning of each year for 20 years. He made his first payment
at the beginning of year 1991 and last payment at the beginning of year 2011. How mush should
be there in his account on 31 Dec. 2011, if the 10% interest rate is compounded annually.

Solution:

10
Given A = Rs. 3000; N = 12 years; R = 10% p.a., implies i  = 0.1
100
Therefore, the accumulated Value V 
A
i
 
(1  i) N  1 1  i 

=
30000
0.1
 
(1  0.1) 20  1 1  0.1

= Rs. 18,90,075

4) A person is repaying a debt with payments of Rs.2,500 per month, at the beginning of every
month. If he misses his payments for July, August, September and October, what payment will
be required in November to put him back on schedule, if interest is at 12.6% per annum?

Solution:

The person has to pay four overdue payments with interest, along with November payment.
Considering only these five payments, we can find the accumulated value to be paid in
November, so that he is put back on schedule.

Given A = Rs. 2,500; N = 5 months; R = 12.5% p.a. =1.05% p.m.,

1.05
which implies i  = 0.0105
100

Therefore, the accumulated Value V 


A
i
 
(1  i) N  1 1  i 

=
2,500
0.0105
 
(1  0.0105) 5  1 1  0.0105

= Rs. 12,900.

5) What amount should be set-aside at the beginning of each year to amount to Rs.38,688.12 at the
end of 10 years at 14% per annum compounded annually?

Solution:

14
Given V = Rs. 38,688.12; N = 10 years; R = 14% p.a, implies i  = 0.14
100

Therefore, the accumulated value V 


A
i
 
(1  i) N  1 1  i 

Substituting the given values, we get

38688.12 
A
0.14
 
(1  0.14)10  1 1  0.14

Hence, the annuity A = Rs. 1755


6) A bank pays 8% per year interest, compounded quarterly. What equal deposits have to be made
at the beginning of each quarter for 10 years, to have Rs. 30,200 at the end of 10 years?

Solution:

Given V = Rs. 30,20; N = 10 years = 40 quarter years;

8 2
R = 8 % p.a.= per quarter year = 2 p.q.y, implies i  = 0.02
4 100

Therefore, the accumulated value V 


A
i
 
(1  i) N  1 1  i 

Substituting the given values, we get

30,200 
A
0.02
 
(1  0.02) 40  1 1  0.02

Hence the annuity A = Rs. 490.20

7) Find the present value of an annuity due of Rs. 1000 per annum for 10 years allowing compound
interest at 5% per annum.

Solution:

5
Given A = Rs. 1,000; N = 10 years; R = 5 % p.a., implies i  = 0.05
100

Therefore, the present value P 


A
i
 N

1  1  i  1  i 

=
1000
0.05
 10

1  1  0.05 1  0.05 = Rs. 8,100.

8) Find the present worth of an annuity due of Rs. 500 payable at the beginning of each month for 2
years, if the money is worth 8% compounded monthly.

Solution:

Given A = Rs. 500; N = 2 years = (2  12) months = 24 months

8 2 2
R = 8 % p.a.= % p.m.= % p.m., implies i  =0.00667
12 3 300

Therefore, the present value P 


A
i
 N

1  1  i  1  i 

500
0.00667
  24

1  1  0.00667  1  0.00667 

= Rs. 31,694.26

9) Find the present value of an annuity due of Rs. 900 payable at the beginning of 6 months for 10
years if the money worth 8% is compounded semi annually.

Solution:

Given A = Rs. 900; N = 10 years = (10  2) half-years = 20 half-years

8
4
R = 8 % p.a.= 2 % per half year = 4% p.h.y., implies i  =0.04
100

Therefore, the present value P 


A
i
 N

1  1  i  1  i 


900
0.04
  20

1  1  0.04 1  0.04

= Rs. 12,720.24

10) Find the present value and amount of an annuity due of Rs. 1500 payable once in the beginning
of two months for 4 years, if the money is worth 12% compounded once in two months.

Solution:

Given A = Rs. 1500; N = 4 years = (4  6) two month periods

= 24 two month periods

12
R = 12 % p.a.= % per two month period
6

= 2% per two month period.

2
Hence, i  = 0.02
100

Therefore, the present value P 


A
i
 N

1  1  i  1  i 


1500
0.02
  24

1  1  0.02 1  0.02

= Rs.28,936.89.
Supplementary Problems
1) Find the amount of an annuity due of Rs. 5750 per annum for 10 years at the rate of interest of
10% per period.
2) Find the amount of an annuity due of Rs. 10000 per annum for 20 years at the rate of interest of
8% per period.
3) Find the amount of an annuity due of Rs. 7600 per annum for 15 years at the rate of interest of
12% per period.
4) Find the amount of an annuity due of 36 monthly payments of Rs. 12,000 that earn interest at
12% per annum compounded monthly.
5) Find the amount of an annuity due of 24 monthly payments of Rs. 5000 that earn interest at 6%
per annum compounded monthly.
6) Find the amount of an annuity due of 18 monthly payments of Rs. 8,500 that earn interest at
14.5% per annum compounded monthly.
7) A person is repaying a debt with payments of Rs.1,000 per month. If he misses his payments for
January, February, March, April and May, what payment will be required in the beginning of
June to put him back on schedule, if interest is at 12% per annum?
8) A person is repaying a debt with payments of Rs.1,650 per month. If he misses his payments for
July, August, September, October and November, what payment will be required in the
beginning of December to put him back on schedule, if interest is at 8.5% per annum?
9) What amount should be set-aside at the beginning of each year to amount to Rs.25,343.82 after 5
years at 8% per annum compounded annually?
10) What amount should be set-aside at the beginning of each year to amount to Rs.35096.96 after 15
years at 7.5% per annum compounded annually?
11) Find the present worth of an annuity due of Rs. 1200 payable at the beginning of each month for
2 years, if the money is worth 18% p.a compounded monthly.
12) Find the present value and amount of an annuity due of Rs. 1500 payable once in the beginning
of two months for 4 years, if the money is worth 12% p.a compounded once in two months.
Answers:

1) Rs. 1,00,804.21 2) Rs. 4,94,235 3) Rs. 3,17,327.36


4) Rs. 5,22,129.60 5) Rs. 1,27,836 6) Rs. 1,71,825.71
7) Rs. 6,213.52 8) Rs. 10,148.16 9) Rs. 4,000
10) Rs. 1250 11) Rs. 24,400.60 12) Rs. 28,939.95.
2.3.5 DEFERRED ANNUITY:
As defined earlier deferred annuity is an ordinary annuity, in which the first payment is
postponed until the expiry of certain time period. If the first payment in an ordinary annuity made at
the end of (k + 1) periods and thereafter payments are made regularly at the end of each period for N
periods, then this ordinary annuity is called a deferred annuity on N periods, deferred by k periods.

AMOUNT OF THE DEFERRED ANNUITY:

The amount of a deferred annuity of N periods deferred k periods at R% per period is given
A
 
by V  (1  i) N  1 , where i 
i
R
100
. Clearly, the amount is independent of deferment period k.
PRESENT WORTH OF A DEFERRED ANNUITY:

The present worth of deferred annuity of k periods at R% per period is given


P
A
i
 
(1  i) k 1  (1  i)  N
.

Solved Problems
1) An annuity consists of 4 annual payments of Rs. 1500 each, the first being made at the end of 6th
year. Find the amount of annuity, if the money is worth 10% effective.

Solution:

The first payment is made at the end of the 6th year. Therefore, the annuity is a deferred annuity.
Given that there are N = 4 annual payments, A = Rs. 1500 per month. Also given that R = 10%
10
p.a., which implies i  =0.1.
100

The amount V of the deferred annuity is given by

V 
A
i
 
(1  i) N  1

=
1500
0.1

(1  0.1) 4  1
= Rs. 6961.50

2) An annuity consists of 30 semi-annual payments of Rs. 500, the first being payable at the end of
4.5 years. Find the amount of the annuity if the money is worth 8% converted semi annually.

Solution:

Here the payment is made at the end of 4.5 years, i.e. 9 half years.

Given that the semi-annual or deferred annuity (A) = Rs. 500

The number of periods N = 30 half years; Rate R = 8 % p.a. = 4% p. h. y.,

4
which implies i  = 0.04. Substituting these values in
100

V 
A
i
 
(1  i) N  1

we get, V 
500
0.04
 
(1  0.04) 30  1 = Rs. 28,042.50.
Hence, the accumulated amount V = Rs. 28,042.50.
3) Find the present value of sequence of annual payment of Rs. 2500 each, the first being made at
the end of 4 years and the last at the end of 10 years, if money is worth 8% per annum.

Solution:

Here the first payment is made at the end of 4th year and the last payment is made at the end of
10th year. Therefore N = 10 – k = 7 years, where the deferred period k = (4-1) years. Also
8
given that the rate R = 8% p.a, which implies i  = 0.08. Substituting these values in
100

P
A
i

(1  i) k 1  (1  i)  N 

we get, P 
2500
0.08
 
(1  0.08) 3 1  (1  0.08) 7 = Rs. 10331.80.

Hence the present value P = Rs. 10,331.80.

4) A land is sold for Rs. 150000 down and 120 monthly payments of Rs. 5000 each, the first being
due 3 years hence. Find the cash value of the house, if money worth 18% per annum,
compounded monthly.

Solution:

Here the first payment is made at the end of 3 years = 36 months.

Therefore, the deferred period of annuity = k = 35 months.

The number of installment months = N = 120 months.

18 R 1 .5
Rate R = 18 % p.a. = =1.5% p.m., which implies i  = = 0.015.
12 100 100

Substituting these values in

P
A
i

(1  i) k 1  (1  i)  N 

we get, P 
5000
0.015
 
(1  0.015) 35 1  (1  0.015) 120 = Rs. 1,64,807.25

Hence the present value P = Rs. 1,64,807.25

Therefore, the cash value of the house = Rs. (164807.25 + 150000)

= Rs. 3,14, 807.25.

5) A man paid his advance taken for house construction in equal installments of Rs.25000 semi
annually in 10 years. If the money was worth 8% per annum compounded semi annually and
payment started after initial gap of 5 years, find the sum of advance taken.

Solution:

Here deferment period of deferred annuity (k) = 9 half years. ( since he starts paying at the end of
5th year)

Number of installments or period (N) = 10 half years.

4
Rate R = 8% per annum = 4% per half year, which implies i  =0.04.
100

Then the present worth P 


A
i

(1  i) k 1  (1  i)  N 
25000
 (1  0.04) 9 (1  (1  0.04) 10 ) =Rs.1,42,452.15
0.04

Supplementary Problems
1) An annuity consists of 3 annual payments of Rs. 500 each, the first being made at the end of 5 th
year. Find the amount of annuity, if the money is worth 12% effective.
2) An annuity consists of 10 annual payments of Rs. 1200 each, the first being made at the end of
3rd year. Find the amount of annuity, if the money is worth 10% effective.
3) An annuity consists of 10 semi-annual payments of Rs. 2000, the first being payable at the end of
1.5 years. Find the amount of the annuity if the money is worth 13% converted semi annually.
4) An annuity consists of 10 quarterly payments of Rs. 1200, the first being payable at the end of
3.5 years. Find the amount of the annuity if the money is worth 11% converted quarterly.
5) Find the present value of sequence of annual payment of Rs. 1250 each, the first being made at
the end of 5 years and the last at the end of 10 years, if money is worth 10% per annum.
6) Find the present value of sequence of annual payment of Rs. 1750 each, the first being made at
the end of 2 years and the last at the end of 5 years, if money is worth 13% per annum.
7) A house is sold for Rs. 1200000 down and 20 half yearly payments of Rs. 2,00000 each, the first
payment paid at the end of the first year. Find the cash value of the house, if money worth 10%
per annum, compounded half-yearly.
8) A house is sold for Rs. 500000 down and 120 monthly payments of Rs. 20000 each, the first
payment paid at the end of the first month of the 3rd year. Find the cash value of the house, if
money worth 9% per annum, compounded monthly.
9) A person borrowed Rs. 85000 at 8% per annum at quarterly compounded interest and agreed to
pay the total money in 36 equal quarterly installments. What should be the installment, if
payment has to start 2 years hence.
10) A sum of money should be set-aside at a child’s birth to provide eight semi annual payments of
Rs. 100000 for a university education, if the first payment is to be made on the child’s 19 th
birthday? The fund will earn interest at 8% per annum.
Answers:

1) Rs. 1,687.20 2) Rs. 19,124.90 3) Rs. 77,650.62

4) Rs. 16,3926.74 5) Rs. 7,970.20 6) Rs. 5,885.34

7) Rs. 11,86,880.90 8) Rs. 35,73,761.76 9) Rs. 3830.30

10) Rs. 4,28,007.53

2.3.6 PERPETUITY:

Perpetuity is an annuity whose payments begins on a fixed date and continues forever.
Examples of perpetuities are the interest payments from a sum of money invested, permanently,
scholarships paid from an endowment on a perpetual basis, and the dividends on a share of preferred
stock (assuming that the company will never become bankrupt).

Let P be the discounted value of an ordinary simple perpetuity, which is an infinite series of
equal payments, made at the ends of interest periods. Let i be the interest rate per period, and
A the periodic payment of the perpetuity. Then, the present value of an ordinary simple perpetuity is
given by

P  A(1  i) 1  A(1  i) 2  A(1  i) 3  


 A (1  i) 1  (1  i) 2  (1  i) 3   
 (1  i) 1 
 A 1 
1  (1  i)  (Using the sum of geometric series)

 1  A
 A 
 (1  i )  1 i .

Solved Problems
1) Find the Present value of an ordinary simple perpetuity paying Rs. 500 a month, if (a) R = 12
% per annum (b) R = 24 % per annum.

Solution:

R
12 12
(a) Given A = Rs. 500; i= = =0.01
100 1200

A 500
Therefore the present value P = = = Rs. 50,000.
i 0.01
R
12 24
(b) Given A = Rs. 500; i= = =0.02
100 1200

A 500
Therefore the present value P = = = Rs. 25,000.
i 0.02

2) How much money is needed to establish a scholarship fund paying Rs. 10000 annually, if the
fund will earn interest at 12% per annum and the first payment be made at the end of the first
year?

Solution:

R 12
Given A = Rs. 10000; i = = =0.12
100 100

A 10000
Therefore the present value P = = = Rs. 83,333.33.
i 0.12

3) A certain stock is expected to pay a dividend of Rs. 10000 at the end of each quarter for an
indefinite period in the future. If an investor wishes to realize an annual effective yield of 12%,
how much should he pay for the stock?

Solution:

Given A = Rs. 10000.

Since the investor wishes to realize an annual effective yield of r = 12 %, we find the actual rate
q
 R 
R r  1   1
4  100q 
of interest per quarter. For that consider, . Substituting r = 0.12, and q = 4
, we get
4
 R 
0.12  1   1
 400 

R 1

Simplifying, interest per quarter = 4 = 100 ( (1  0.12) 4


 1 ) = 2.87374 %

R
i  0.0287374
or 400 .

A 10000
Therefore, the present value P = i = 0.0287374 = Rs.3,47,978.59.

4) A scholarship fund is established by an endowment of Rs. 2,00,00,000 invested at 10% per


annum compound interest. What payment will this fund provide
(a) at the end of each year, forever?

(b) at the end of every 2 years, forever?

Solution:

R 10
i 
Given P = Rs. 2,00,00,000 and 100 100 =0.1

A
P
(a) Since i , we have A  Pi . Therefore, A = 20000000 (0.1)

= Rs. 20,00,000.

Therefore, the payment the fund provide = Rs. 20,00,000 per year

A
P
(b) Since 2i , we have A  P(2i) . Therefore, A = 20000000 (0.2)

= Rs. 40,00,000.

Therefore, the payment the fund provide = Rs. 40,00,000 per year.

Supplementary Problems
1) Find the Present value of an ordinary simple perpetuity paying Rs. 1,500 a month, if (a) R =
10 % per annum (b) R = 18 % per annum.
2) Find the Present value of an ordinary simple perpetuity paying Rs. 2000 per quarter, if (a) R =
12 % per annum (b) R = 36 % per annum.
3) How much money is needed to establish a scholarship fund paying Rs. 1,00,000 annually, if the
fund will earn interest at 18% per annum and the first payment be made at the end of the first
year?
4) How much money is needed to establish a scholarship fund paying Rs. 5,00,000 annually, if the
fund will earn interest at 24% per annum and the first payment be made at the end of the first
year?
5) A certain stock is expected to pay a dividend of Rs. 50000 at the end of each quarter for an
indefinite period in the future. If an investor wishes to realize an annual effective yield of 8%,
how much should he pay for the stock?
6) A certain stock is expected to pay a dividend of Rs. 75,000 at the end of each quarter for an
indefinite period in the future. If an investor wishes to realize an annual effective yield of 10%,
how much should he pay for the stock?
7) A certain stock is expected to pay a dividend of Rs. 10000 at the end of every six months for an
indefinite period in the future. If an investor wishes to realize an annual effective yield of 12%,
how much should he pay for the stock?
8) A certain stock is expected to pay a dividend of Rs. 25000 at the end of each month for an
indefinite period in the future. If an investor wishes to realize an annual effective yield of 15%,
how much should he pay for the stock?
9) A scholarship fund is established by an endowment of Rs. 50,00,000 invested at 5% per annum
compound interest. What payment will this fund provide (a) at the end of each year, forever?
(b) at the end of every 2 years, forever?
Answers:

1) (a) Rs. 1,80,000 1) (b) Rs. 1,00,000 2) (a) Rs. 66,666.67


2) (b) Rs. 22,000.22 3) Rs. 5,55,555.56 4) Rs. 20,83,333.30
5) Rs. 25,73,797.60 6) Rs. 31,10,266.50 7) Rs. 1,71,525.04
8) Rs. 21,34,031.40 9) (a) Rs. 2,50,000 9) (b) Rs. 5,00,000

2.3.6 SINKING FUND:

If an accumulated amount of annuity to be used for the replacement of the wasted asset such
as plant machinery etc. or to be used for liquidation of loan or to be used for paying off financial
obligations at some future pre decided date or for the expansion of business.

The amount of a sinking fund at any time is the amount of the annuity formed by the periodic
deposits or payments. If ‘a’ is the periodic deposits or payment, n is the number of periods of
payments and R% be the interest per period, then the amount in a sinking fund is given by

 (1  i ) N  1 
F  A  
 i ,

R
where i  .
100

The periodic payment or deposit required to accumulate an amount of Rs. F over N periods at
the rate of interest R% per period is given by

F i
A
 
(1  i) N  1 ,

R
where i  .
100

Solved Problems

1) A firm anticipates a capital expenditure of Rs. 2,50,000 for new equipment in 5 years. How
much should be deposited annually in a sinking fund carrying 12% per annum interest
compounded annually to provide for a purchase.
Solution:

Given F = Rs. 2,50,000; N = 5 years; R = 12 % p.a.

12
i = 0.12
100

F i 250000  0.12
Therefore, the annuity A 
   
=
(1  i)  1
N
(1  0.12) 5  1

30000
=
1.76234  1

30000
= = Rs. 39,355.
0.76

2) A company sets aside a sum of Rs. 30,000 at the end of each year for 20 years in a sinking fund,
which earns 12 % compound interest. The balance amount sufficient to replace equipment. Find
the expected value of the equipment at that time.

Solution:

Given A = Rs. 30,000; N = 20 years; R = 12 % p.a.

12
i = 0.12
100

Therefore, the expected value of the equipment at that time is

 (1  i ) N  1   (1  0.12) 20  1 

F  A 
 = 30000  
 i   0.12 

 9.6463  1 
= 30000  
 0.12 

= 30000 72.0525 = Rs. 21,61,575.

3) A company decides to create a sinking fund for paying the debenture issue of Rs.1,00,000 at the
end of 12 years. Find the sum retained out of the profit at an end of each year, assuming the
money accumulated earns 8% per annum interest, compounded annually.

Solution:

Given F = Rs. 1,00,000; N = 12 years; R = 8 % p.a.


8
i = 0.08
100

F i 100000  0.08
Therefore, the annuity A 
   
=
(1  i)  1
N
(1  0.08)12  1

8000
=
2.5182  1

30000
= = Rs. 19,760.24.
1.5182

4) What sum should be set aside biannually into an account paying 9% per annum interest
compounded half yearly, if it has to buy a machine expected to cost Rs. 2,00,000 at the end of 8
years.

Solution:

Given F = Rs. 2,00,000; N = 8 years; R = 9 % p.a.

9
i = 0.09
100

F i 200000  0.09
Therefore, the annuity A 
   
=
(1  i)  1
N
(1  0.09) 8  1

18000
=
1.9926  1

18000
= = Rs. 18,134.20.
0.9926

5) A company establishes a sinking fund for payment of Rs. 1,00,00,000 debt maturing in 8 years.
How much should be deposited quarterly into sinking fund carrying 15 % per annum
compounded quarterly to provide for the purpose.

Solution:

Given F = Rs. 1,00,00,000; N = 8 years = 24 quarters

R = 15 % p.a. = 3.75 % per quarter

3.75
i = 0.15
100
F i 10000000  0.0375
Therefore, the annuity A 
   
=
(1  i)  1
N
(1  0.0375) 24  1

375000
=
3.2480  1

375000
= = Rs. 1,66,814.95
2.2480

6) A company creates a sinking fund for the redemption of debentures of Rs. 1,60,000 at the end of
16 years. If the company invests Rs. 60,000 at the end of each year in the sinking fund and the
fund accumulates 8% per annum, compounded annually. Find the extra money in the fund after
paying off the debentures.

Solution:

Given A = Rs. 60,000; N = 16 years; R = 8 % p.a.

8
i = 0.08
100

Therefore, the accumulated value is

 (1  i ) N  1   (1  0.08)16  1 
F  A   = 60000  
 i   0.08 

 3.42594  1 
= 60000  
 0.08 

= 60000 30.32425 = Rs. 18,19,475.

The amount paid off as debentures = Rs. 16,00,000

Hence, the extra money in the fund after paying off the debentures

= Rs. (18,19,475 – 16,00,000)

= Rs. 2,19,455

Supplementary Problems
1) A firm anticipates a capital expenditure of Rs. 1,40,000 for new equipment in 10 years. How
much should be deposited annually in a sinking fund carrying 10% per annum interest
compounded annually to provide for a purchase.
2) A company sets aside a sum of Rs. 15,000 at the end of each year for 15 years in a sinking fund,
which earns 8 % compound interest, the balance amount sufficient to replace equipment. Find
the expected value of the equipment at that time.
3) A company decides to create a sinking fund for paying the debenture issue of Rs.2,50,000 at the
end of 20 years. Find the sum retained out of the profit at an end of each year, assuming the
money accumulated earns 9% per annum interest, compounded annually.
4) What sum should be set aside biannually into an account paying 13% per annum interest
compounded half yearly, if it has to buy a machine expected to cost Rs.40,00,000 at the end of 5
years.
5) A company establishes a sinking fund for payment of Rs. 25,00,000 debt maturing in 10 years.
How much should be deposited quarterly into sinking fund carrying 10 % per annum
compounded quarterly to provide for the purpose.
6) A company establishes a sinking fund for payment of Rs. 5,00,000 debt maturing in 5 years.
How much should be deposited half-yearly into sinking fund carrying 6% per annum
compounded half-yearly to provide for the purpose.
7) A company establishes a sinking fund for payment of Rs. 3,50,000 debt maturing in 7 years.
How much should be deposited monthly into sinking fund carrying 11 % per annum compounded
monthly to provide for the purpose.
8) A company creates a sinking fund for the redemption of debentures of Rs.2,00,000 at the end of
10 years. If the company invests Rs. 1,00,000 at the end of each year in the sinking fund and the
fund accumulates 6% per annum, compounded annually. Find the extra money in the fund after
paying off the debentures.
9) A company creates a sinking fund for the redemption of debentures of Rs.10,00,000 at the end of
5 years. If the company invests Rs. 2,00,000 at the end of each year in the sinking fund and the
fund accumulates 9% per annum, compounded annually. Find the extra money in the fund after
paying off the debentures.
10) A company creates a sinking fund for the redemption of debentures of Rs.70,000 at the end of 3
years. If the company invests Rs. 5,000 at the end of each year in the sinking fund and the fund
accumulates 12% per annum, compounded monthly. Find the extra money in the fund after
paying off the debentures.

Answers:
1) Rs. 8,784.60 2) Rs. 4,07,287.50 3) Rs. 4,886.63
4) Rs. 6,17,284 5) Rs. 37,090.60 6) Rs. 43,615.25
7) Rs. 2,784.52 8) Rs. 11,18,079.50 9) Rs. 1,96,942.12
10) Rs. 5384.54
3
Unit

RATIO AND PROPORTIONS

3.1 RATIO

atio is a relation / comparison of two numbers by division. i.e, ratio of


R one quantity to another is the relation in respect of number of times that
one contains the other.
A Ratio is the relationship between two quantities of same kind, with respect to
magnitude and denotes how many times one of the quantity is contained in the
other.
a
The ratio of two quantities ‘a’ and ‘b’ is written as a : b or .
b
The first term of the ratio is called the antecedent and second term is called the
consequent.

Example:
1. Suppose Mr. A is getting a salary of Rs.15000/- per month and Mr.B is
getting a salary of Rs.5000/- per month. Find the ratio of their salaries?

Salary of Mr.A 15000 3


Sol:  
Salary of Mr.B 5000 1
 salary of Mr.A is thrice that of Mr.B.
Remark:
1. Quantities of same kind can be expressed as ratio, but dissimilar quantities
cannot.

2. Ratio should always be written in its lowest terms.


3. When two magnitudes of same kind are being compared, they must be
expressed in terms of a common unit of measurement before deriving a ratio.

4. A ratio remains unaltered if its numerator and denominator are multiplied or


divided by same number.

5. When a certain quantity Q is to be divided in a given ratio a:b, the two parts
aQ bQ
are and .
ab ab
6. In ratios, order of elements is important.

Eg:- If there are 18 ducks and 7 geese in a park. The ratio of geese to duck is
7: 18 and ratio of duck to geese is 18:7.
7. To compare two or more ratios, we write the ratios in terms of fractions and
make their denominators equal by necessary mathematical operations (eg:-
LCM). Then comparing their numerators we decide which one is greater
than/less than/equal to other.

Eg:- Find which is the larger ratio among, 4:10 and 3:7
Sol: LCM of 10 and 7 is 70.
4 4*7 3 3 *10
  and 
10 10 * 7 7 7 *10
28 30 30
i.e. and . Comparing the numerators we get is larger, which
70 70 70
correspond to the ratio 3:7.
Compound Ratio: For ratios a : b and c : d, the compound ratio is ac : bd.
Duplicate Ratio: For ratio a : b, its duplicate ratio is a2 : b2.
Sub duplicate Ratio: For ratio a : b, its sub duplicate ratio is a : b .
Triplicate Ratio: For ratio a : b, its triplicate ratio is a3 : b3 .
Sub triplicate Ratio: For ratio a : b, its sub triplicate ratio is 3 a : 3 b .
Reciprocal ratio: For ratio a : b, where a  0, b  0 its reciprocal ratio is
1 1
:  b :a .
a b

PRACTICE PROBLEMS:
1. A’s monthly salary is Rs. 250 and B’s annual salary is Rs.4000. What is the
ratio of their income?
A' s annual income 12 * 250
Sol:  =3:4.
B' s annual income 4000
2. Divide 81 in the ratio 2:7?

2 7
Sol: First part = 81 *  18 . Second part = 81 *  63 .
9 9
3. If x : y =2 : 3,find (x2y + xy2) : (x3 + y3).

2y
Sol: x 
3
x 2 y  xy 2
Substituting in we get
x 3  y3
 2y  2  4y y   2y y 2  4   2 
2
 2y 
2

  y   y       
 3   3   9   3 
      6 : 7.
9 3
 
 8y   8 
3
 2y 
3

  y  y   1
3 3

 3   27   27 
4. Two numbers are in the ratio 3: 5. If 8 is added to each number then the ratio
becomes 2:3. Find the numbers.

Sol: Let the numbers be 3x and 5x.


3x  8 2
Given, 
5x  8 3
i.e., 3 (3x+8) = 2(5x+8)
i.e., 9x+24 = 10x+16
 x=8.
Numbers are 24 and 40.
5. Two numbers are in the ratio 8:9. If sum of the numbers is 119, find the
numbers.

Sol:
Method –I Method-II
Given problem is equivalent to divide Let the two numbers be 8x and 9x.
119 in to two parts in the ratio 8:9. Given, sum of two numbers = 119.
8 i.e., 8x+9x =119 i.e., 17x =119
first number = *119  56 i.e., x=7
89
the numbers are 8*7 =56 and
9*7=63.
9
Second number = *119  63
89

6. The ratio of men and women workers in a factory is 5:3. The rate of daily
wages payable to them is in the ratio of 3:2. Find the ratio of total wages
paid to them for a week.

Sol: Ratio of men and women is 5:3


Ratio of daily wages is 3:2
Ratio of total wages per day is 5*3 : 3*2 = 15 : 6.
Ratio of wages per week is 15 *7 : 6 *7 = 5 : 2.
7. Divide Rs. 510 between A, B and C so that A gets 2/3 of what B gets and B
gets ¼ of what C gets. Find the share of each?

1 2 1
Sol: Let share of C be x. Share of B = x . Share of A = * x .
4 3 4
1 1
Ratio of share’s of A, B and C is : : 1  2 : 3 :12.
6 4
2
A’s share = 510 *  Rs.60 ,
17
3
B’s share = 510 *  Rs.90 ,
17
12
C’s share = 510 *  Rs.360 .
17
8. Find the ratio compounded of duplicate ratio of 5:6, reciprocal ratio of 25:42
and sub triplicate ratio of 216: 343.

Sol: Duplicate ratio of 5:6 is 25: 36


Reciprocal ratio of 25: 42 is 42: 25
Sub triplicate ratio of 216: 343 is 6: 7.
 Compound ratio = (25*42*6) : (36* 25* 7) = 1:1
9. Compare the following ratios 4:5, 7:8 and 6:10. Find the greatest and least
ratios.

Sol: LCM (5, 8, 10) = 40.


4 * 8 32
4:5  
5 * 8 40
7 * 5 35
7 :8  
8 * 5 40
6 * 4 24
6 : 10  
10 * 4 40
Comparing the numerators we find that greatest ratio is 7:8 and least ratio is
6:10.
1
10.The adulterated milk of 20 L contains 6 L of pure milk. Find the ratio of
4
pure milk to water in the mixture.

25 25 55
Sol: Quantity of pure milk = , Quantity of water = 20  
4 4 4
25 55
Ratio of pure milk to water = : = 5 : 11.
4 4
11.Train A covers 144kms of distance in 1.2 hours and train B takes 1.8 hours.
Find the ratio of their speed per hour.

Sol: Speed = distance/ time.


144
Speed of train A =  120 km/hr.
1.2
144
Speed of train B =  80 km/hr
1.8
120
Ratio of speed of train A and B =  3: 2.
80
12.Monthly incomes of A and B are in the ratio 3 : 4 and their monthly savings
are in the ratio 1 : 2. If each spends Rs. 2000, find their monthly income and
savings.

Sol: Savings = Income – Expenditure


Savings of A 3x  2000

Savings of B 4x  2000
3x  2000 1
i.e., 
4x  2000 2
i.e., 2(3x – 2000) = 1(4x – 2000)
i.e., x= 1000.
Income and savings of A are Rs. 3000 and Rs 1000 respectively.
Income and savings of B are Rs. 4000 and Rs 2000 respectively
13. A man gave 3/8 of a sum of money to his I son and gave 30% of the
remaining amount to the II son. He then gave the remaining sum of money
to three beggars in the ratio 2: 5: 7. The difference in his son’s share is Rs.
4200. What was the sum of money and how much was given away to
beggars?

Sol: Let sum of money be Rs. X.


I son share = 3x/8. Remaining amount = x- 3x/8 =5x/8
II son’s share = .30 *5x/8 = .1875x
Given 3x/8 - .1875x = 4200
i.e., x =22400.
Money given to beggars = x- (3x/8 +0.1875x)
=.4375x
=Rs.9800
2
Share of I beggar = * 9800  Rs.1400
14
5
Share of II beggar = * 9800  Rs.3500
14
7
Share of III beggar = * 9800  Rs.4900
14
Exercise:
1. Find the ratio compounded of (i) 2 : 3 and 6 : 7, (ii) 3 : 5, 7 : 9 and 10 : 21.

2. Find the duplicate and triplicate ratio of 3 : 5.

3. Find the sub duplicate and sub triplicate ratio of 64 : 729.

4. Find the ratio compounded of 4 : 5, duplicate ratio of 2 : 3 and sub duplicate


ratio of 81 : 225.

5. Find the value of x, if (3x+4) : (4x+9) is the duplicate ratio of 4 : 5.

6. Divide Rs. 72 in the ratio 4 : 5.

7. Two quantities are in the ratio 3:4 and if 10 is subtracted from each of them,
the remainders are in the ratio 1:3. What are the quantities?

8. If a : b = 2 : 3 and x : y = 4 : 5. Find the value of (5ax+3by) : (10ax+4by).


9. If x : y = 4 : 3, find the value of (4x+3y) : (7x+6y).

10.The ratio of number of boys and girls in a school is 2 : 5. If there are 350
students in the school, find the number of girls in the school.

11.A sum of money is divided between A and B in the following manner: One-
sixth is paid to A and a eighth to B; then 5/9 of the remaining amount again
to A and the rest to B. If A received Rs. 2420, what does B receive?

3.2 PROPORTION
A statement (or an equation) which represents that two ratios are equal is called a
a c
proportion. If  then a, b, c and dare said to be in proportion, and is denoted
b d
as a : b = c : d or a : b :: c : d.
1. The terms a and d are called extremes and b and c are called means.

2. d is called the fourth proportional to a, b and c.

3. Product of means = product of extremes. i.e., bc = ad.

Continued Proportion
If a : b = b : c, then a, b and c are said to be in continued proportion. In this case c
is called the third proportional of a and b and b is called the mean proportional of a
and c.
RULES OF PROPORTION
1. If a : b = c : d, then b : a = d : c. (Invertendo rule).

2. If a : b = c : d, then a : c = b : d. (Alternendo rule).

3. If a : b = c : d, then a+b : b = c+d : d. (Componendo rule).

4. If a : b = c : d, then a-b : b = c-d : d. (Dididendo rule).

5. If a : b = c : d, then a+b : a-b = c+d : c-d. (Componendo- dividendo rule).


Remark
If ratio between two quantities Q1 and Q2 is a1 : a2, ratio between two
quantities Q2 and Q3 is b2 : b3, and ratio between two quantities Q3 and Q4 is c3 : c4,
b3 c b
then ratio between Q1, Q2, Q3 and Q4 is a1 : a2 : a2: 4 3 a2 .
b2 c3 b 2
PRACTICE PROBLEMS:

1. Find the fourth proportional of the following: Rs. 48, Rs. 36 and 4 meters.

Sol: Let x meters be the fourth proportional.


 48 : 36 = 4 : x
48 4 4 * 36
i.e.,  , x  = 3.
36 x 48
Fourth proportional = 3 meters.
2. If 3, x, 1083 are in continued proportion, find x.

Sol: 3 : x = x : 1083
3 x
i.e.,   x2 = 3* 1083  x  3249  57.
x 1083
3. If x : y = 5 : 3 and y : z = 5 : 6 then find the ratio of x : y :z.

6
Sol: x : y : z = 5 : 3 : *3
5
18
=5:3:
5
= 25 : 15: 18.
4. If a+b : a-b =7 : 3, find the value of a : b.

ab 7
Sol:   3a  3b  7a  7b  10b = 4a
ab 3
a 10 a 5
    .
b 4 b 2
ax  ax
5. Solve  a.
ax  ax

ax  ax a
Sol: Let 
ax  ax 1
Applying componendo-dividnedo rule we get,

ax  ax  ax  ax   
a 1 

ax  ax  ax  ax   a 1 
2 a  x a  1 a  x a  1
2

   
2 a  x a  1 a  x a  1
2
Applying componendo-dividnedo rule we get,
(a  x )  (a  x ) a  1  a  1
2 2


(a  x )  (a  x ) a  1  a  1
2 2

2a (a 2  2a  1)  (a 2  2a  1)
 
2x (a 2  2a  1)  (a 2  2a  1)
a 2(a 2  1) a a2 1 x 2a 2a 2
      2 x 2 .
x 2(2a ) x 2a a a 1 a 1
6. The ratio of number of boys and girls in a school is 4 : 3. If there are 480
boys in the school, find the number of girls in the school.

Sol: 4: 3 = 480 : x.
480 * 3
x   360 girls .
4
7. In a film shooting A and B received amount in a certain ratio. B and C
received the amount in the ratio of A and B. If A gets Rs. 1,60,000/- and C
gets Rs.2,50,000. Find the amount received by B?

Sol: Given Ratio of amount received by A and B = Ratio of amount


received by B and C
160000 x
i.e,   x 2  160000 * 250000  Rs.200000 / 
x 250000
8. In 100 meters running race A beats B by 5.6 meters and B beats C by 12.5
meters. By how much does A beat C?

Sol: Distance covered by A=100m


Distance covered by B = 100- 5.6 = 94.4m
Ratio of distance covered by A and B is 100 : 94.4
Distance covered by B=100m
Distance covered by C = 100- 12.5 = 87.5m
Ratio of distance covered by B and C is 100 : 87.5
87.5
Hence, ratio of distance covered by A, B and C is 100 : 94.4 : * 94.4
100
i.e., 100 : 94.4 : 82.6.
A beats C by 100-82.6=17.4m
9. A concrete mixture was made of cement, chips and sand. The ratio of
cement and chip is same as that of chips and sand. If 100 bags cement and
900 bags of sand were used in the mixture, find the number of bags that
were used in the mixture.

Sol: Let x be the number of bags of chips.


Given, 100: x = x : 900
 x  900 *100  300 .
No: of bags of chips used in the mixture = 300.

Exercise:

1. If 5 : 20 :: 3: x, find x.

2. Find x, if 2 : 3 :: x : 6.

3. Find fourth proportional of 5, 4, 20.

4. Find the third proportional of 8, 40.

5. Find the mean proportional of 7 and 28.

6. The sum of first and third proportional is 130. The middle term is 16. Find the
numbers.

7. What number must be subtracted from each of 9, 11, 15 and 19 so that the
differences will be proportional?

x  12a  x a 1
8. Solve  .
x  12a  x a 1

1 x2  1 x2
9. Solve for x, if  3.
1 x  1 x
2 2

10.If , then prove that .

11.If , find the value of .


12.In 1km running race, A beats B by 50m and B beats C by 125m, by how much
does A beat C?

Direct proportion
A direct proportion means a positive correlation between two related terms
whereby an increase (or decrease) in the value of one is followed by a
proportionate increase (or decrease) in the value of another.
Eg:- Ratio between cost and quantity of an article.
Inverse proportion
A inverse proportion means a negative correlation between two related terms
whereby an increase (or decrease) in the value of one is followed by a
proportionate decrease (or increase) in the value of another.
Eg:- Ratio between number of workers and amount of time.( More workers means
less time will be required to do a work)
Ratio between supply and price.
Compound Proportion
A proportion that relates more than two ratios is called a compound proportion.
If a : b is a ratio which is proportional to c : d and e : f in such a way that ace : bdf
then a : b, c : d and e : f are said to be compound proportions.

PRACTICE PROBLEMS:
1. If 15 chairs cost Rs. 750, what will be the cost of 120 chairs at the same
price?

No : ofchairs Cost
Sol: 15 750
120 x
As number of chairs increases, the total cost also increases. i.e., direct
proportion.
15: 120 = 750 : x
15 750 120 * 750
  x  Rs.6000
120 x 15
2. It takes 3 hours to pump out 2500 litres of water with a certain pump. What
time will it take to pump out 15000 litres.

Hours Litres
Sol: 3 2500
x 15000
Direct proportion, since increase in quantity of water will increase the
number of hours.
3 2500 3 *15000
  x  18hrs.
x 15000 2500
3. 500 workers can finish a work in 8 days. How many workers will finish the
same work in 2 days.

Wor ker s Days


Sol: 500 8
x 2
Inverse proportion, since decrease in number of days will increase the
number of workers to finish the job.
500 2 500 * 8
  x  2000 wor ker s.
x 8 2
4. If 20 men can do a job in 18 days, how long will 60 men take to do the same
job?

Sol: More men  less days to complete the job. i.e., inverse proportion
Men Days
20 18 i.e., 18 : x =60 : 20
60 x
18 60 18 * 20
  x  6 days.
x 20 60
5. If Rs.120 is used to maintain a family of 4 persons for 30 days, how long is
Rs.204 used to maintain a family of 6 persons.

days persons money


Sol: 30 4 120
x 6 204
As the money increases, the number of days also increases, hence direct
proportion.
As the number of persons increases, the number of days decreases, hence
inverse proportion.
120 : 204
30 : x    30 : x  120 * 6 : 204 * 4
 6 : 4
30 * 204 * 4
x  34days.
120 * 6
6. 15 men work 8 hours a day to complete a piece of work in 25 days. How
many days will be taken by 12 men for doing a similar assignment working
10 hours a day?

Days Men Hours


Sol: 25 15 8
x 12 10
More men  less days. i.e., Inverse proportion
More hours  less days i.e., Inverse proportion
12 : 15
25 : x    25 : x  12 *10 : 15 * 8
 10 :8
25 *15 * 8
x  25 days.
12 *10
7. A contractor undertakes to dig a canal 12kms long in 350 days, and employs
45 men for the same. After 200 days of working he finds that only 4.5kms of
canal has been completed. How many extra men must be employed to finish
the work in time?

Men Days Length


Sol: 45 200 4.5
x 150 7.5
Less days  more men. i.e., inversely proportional
More length  more men, i.e., directly proportional
150 : 200
45 : x    45 : x  150 * 4.5 : 200 * 7.5
 4.5 : 7.5
45 * 200 * 7.5
x  100
150 * 4.5
Extra men employed = 100-45 = 55.
8. 4 men or 12 women can do a job in 5days by working 8 hours per day. In
how many days 2 men and 4 women can do the same job by working 10
hours per day?
Sol: We have to establish equivalence between different categories of
workmen.
Given, Work done by 4 men =work done by 12 women
i.e., Work done by 1 man =work done by 3 women
work done by 2 men and 4 women = work done by (6+4)=10 women.
Days Hours Women
5 8 12
x 10 10
Less women  more days. i.e., inversely proportional
More time  less days, i.e., inversely proportional
10 : 12
5: x    5: x  10 *10 : 12 * 8
 10 : 8
5 *12 * 8
x  4.8 days.
100
9. 10 men or 15 women or 20 children can do a piece of work in 15 days. How
long will 9men, 3 women and 8 children take to do the same piece of work?

Sol: Work done by 10 men= work done by 15 women


10
i.e., work done by men = work done by 1 woman.
15
Work done by 10 men =work done by 20 children
10
i.e., work done by men = work done by 1child.
20
Work done by 9 men,3 women and 8 children = work done by
10 10
9  3*  8*  9  2  4  15men
15 20
Days Men
15 10
x 15
More men  less days. i.e., inversely proportional
15 : x  15 : 10
15 *10
x  10 days.
15
10.3 men and 2 boys can finish a piece of work in 6 days. 5men and 7 boys can
finish the same work in 3 days. In how many days 2 men and 5 boys will
complete the same work?

Sol: 3 men and 2 boys can finish a work in 6 days.


i.e. in 1 day, 18 men and 12 boys can finish the work.
5 men and 7 boys can finish a work in 3 days.
i.e., in 1 day, 15 men and 21 boys can finish the work.
 Work done by 18 men and 12 boys = work done by 15 men and 21 boys.
i.e., work done by 3 men=work done by 9 boys.
i.e., work done by 1 man = work done by 3 boys.
 Work done by 3 men and 2 boys =work done by 11 boys.
Work done by 5 men and 7 boys = work done by 22 boys.
Work done by 2 men and 5 boys = work done by 11 boys.
Days Boys Days Boys
6 11 322
x 11 x11
Since, number of boys are same. Less boys  more days. i.e., inversely
Required number of days = 6. proportional
3: x  11 : 22
3 * 22
x  6 days.
11

Exercise:
1. If 15 men working 12 hours per day, perform job in 16 days, how long will
it take 21 men working 10 hours daily to do the same task.

2. If 10 masons working 7 hours daily can build 8 water tanks in 5 days, how
many days will 15 masons working 3 hours daily take to build 36 water
tanks?

3. If 20 men can dig a tank 30 meters long, 20 meters wide and 12 meters deep
in 25 days working 8 hours a day, how long will it take to dig three tanks
each 90 meters long, 25meters wide, 20 meters deep using 80 men working
8 hours a day?

4. A typist with a speed of 38 words a minute undertakes to copy a manuscript


of 500 pages in 45 days. At the end of 20 days he finds that working 4 hours
a day he has been able to complete only 150 pages. He calls in a friend with
a speed of 60 words a minute to finish the work in time. How hard sgould
they both work to be within time?

5. A contractor engaged 150 labourers to raise an embankment 27 miles long in


90 weeks. The men are to work 8 hours a day for 6 days a week. At the end
of 60 weeks it was found that 18 miles of work had been finished. How
should the gang be reduced, if at all, to finish the work in time, if the men
put in 9 hours of work a day, all days of a week?

6. 2 men and 4 boys can do a work in 33 days and 3 men and 5 boys can do the
same work in 24 days. How long will 5 men and 2 boys take to finish the
work?

TIME and WORK

1. Mr. A can do a job in 8 days; Mr. B can do it in 5 days. A and B together


undertook to do it for Rs.8000. with the help of an assistant, they completed
the job in 3 days. How should they divide their remuneration?

Sol: Given, A can do a job in 8 days.  In 1day A can do 1 th of the job.


8
 In 3days A can do 3 th of the job.
8
Given, B can do a job in 5 days.  In 1day B can do 1 th of the job.  In
5
3days A can do 3 th of the job.
5
3 3 39
 A and B together can do   of the job in three days.
8 5 40
39 1
Assistant can do 1   of the job in three days.
40 40
They can share their remuneration as follows:
A will get 3 of Rs.8000 = Rs.3000
8
B will get 3 of Rs.8000 = Rs.4800
5
C will get 1 of Rs.8000 = Rs.200
40
2. A and B can do a piece of work in 12 days, B and C in 16 days and C and A
in 24 days. How long will it take for them to do the job if they work
together? How long will it take if each does the job separately?

Sol: In 1 day, A+B can do 1 of the job, B+C can do 1 and A+C can do
12 16
1 of the job.
24
1 1 1 1 3
in 1 day, A+B+C can do      of the job.
 12 16 24  2 32
32
 A, B and C can together finish the entire job in days =10.67days.
3
Work done by A alone in 1 day =work done by A+B+C – Work done by B+C
3 1 1
   .
32 16 32
Work done by B alone in 1 day =work done by A+B+C – Work done by A+C
3 1 5
   .
32 24 96
Work done by C alone in 1 day =work done by A+B+C – Work done by A+B
3 1 1
   .
32 12 96
96
Hence, A, B and C can complete the job separately in 32 days, days and
5
96 days respectively.

You might also like